OB Exam 2

Ace your homework & exams now with Quizwiz!

When assessing a woman in the first stage of labor, which clinical finding will alert the nurse that uterine contractions are effective? a. Dilation of the cervix b. Descent of the fetus to -2 station c. Rupture of the amniotic membranes d. Increase in bloody show

A The vaginal examination reveals whether the woman is in true labor. Cervical change, especially dilation, in the presence of adequate labor, indicates that the woman is in true labor. Engagement and descent of the fetus are not synonymous and may occur before labor. ROM may occur with or without the presence of labor. Bloody show may indicate a slow, progressive cervical change (e.g., effacement) in both true and false labor.

The laboratory results for a postpartum woman are as follows: blood type, A; Rh status, positive; rubella titer, 1:8 (enzyme immunoassay [EIA] 0.8); hematocrit, 30%. How should the nurse best interpret these data? a. Rubella vaccine should be administered. b. Blood transfusion is necessary. c. Rh immune globulin is necessary within 72 hours of childbirth. d. Kleihauer-Betke test should be performed.

A This client's rubella titer indicates that she is not immune and needs to receive a vaccine. These data do not indicate that the client needs a blood transfusion. Rh immune globulin is indicated only if the client has an Rh-negative status and the infant has an Rh-positive status. A Kleihauer-Betke test should be performed if a large fetomaternal transfusion is suspected, especially if the mother is Rh negative. However, the data provided do not indicate a need for performing this test.

The induction of labor is considered an acceptable obstetric procedure if it is in the best interest to deliver the fetus. The charge nurse on the labor and delivery unit is often asked to schedule clients for this procedure and therefore must be cognizant of the specific conditions appropriate for labor induction. What are appropriate indications for induction? (Select all that apply?) a. Rupture of membranes at or near term b. Convenience of the woman or her physician c. Chorioamnionitis (inflammation of the amniotic sac) d. Postterm pregnancy e. Fetal death

A, C, D, E The conditions listed are all acceptable indications for induction. Other conditions include intrauterine growth restriction (IUGR), maternal-fetal blood incompatibility, hypertension, and placental abruption. Elective inductions for the convenience of the woman or her provider are not recommended; however, they have become commonplace. Factors such as rapid labors and living a long distance from a health care facility may be valid reasons in such a circumstance. Elective delivery should not occur before 39 weeks of completed gestation.

What is a maternal indication for the use of vacuum-assisted birth? a. Wide pelvic outlet b. Maternal exhaustion c. History of rapid deliveries d. Failure to progress past station 0

B A mother who is exhausted may be unable to assist with the expulsion of the fetus. The client with a wide pelvic outlet will likely not require vacuum extraction. With a rapid delivery, vacuum extraction is not necessary. A station of 0 is too high for a vacuum-assisted birth.

A nurse is observing a family. The mother is holding the baby she delivered less than 24 hours ago. Her husband is watching his wife and asking questions about newborn care. The 4-year-old brother is punching his mother on the back. How should the nurse react to this situation? a. Report the incident to the social services department. b. Advise the parents that the toddler needs to be reprimanded. c. Report to oncoming staff that the mother is probably not a good disciplinarian. d. Realize that this is a normal family unit adjusting to a major family change.

D The observed behaviors are normal variations of a family adjusting to change. Reporting this one incident is not needed. Offering advice at this point would make the parents feel inadequate.

A client at 39 weeks of gestation has been admitted for an external version. Which intervention would the nurse anticipate the provider to order? a. Tocolytic drug b. Contraction stress test (CST) c. Local anesthetic d. Foley catheter

A A tocolytic drug will relax the uterus before and during the version, thus making manipulation easier. CST is used to determine the fetal response to stress. A local anesthetic is not used with external version. Although the bladder should be emptied, catheterization is not necessary.

The nurse should be cognizant of which important information regarding nerve block analgesia and anesthesia? a. Most local agents are chemically related to cocaine and end in the suffix -caine. b. Local perineal infiltration anesthesia is effective when epinephrine is added, but it can be injected only once. c. Pudendal nerve block is designed to relieve the pain from uterine contractions. d. Pudendal nerve block, if performed correctly, does not significantly lessen the bearing-down reflex.

A Common agents include lidocaine and chloroprocaine. Injections can be repeated to prolong the anesthesia. A pudendal nerve block relieves pain in the vagina, vulva, and perineum but not the pain from uterine contractions. A pudendal nerve block lessens or shuts down the bearing-down reflex.

Which stage of labor varies the most in length? a. First b. Second c. Third d. Fourth

A The first stage of labor is considered to last from the onset of regular uterine contractions to the full dilation of the cervix. The first stage is significantly longer than the second and third stages combined. In a first-time pregnancy, the first stage of labor can take up to 20 hours. The second stage of labor lasts from the time the cervix is fully dilated to the birth of the fetus. The average length is 20 minutes for a multiparous woman and 50 minutes for a nulliparous woman. The third stage of labor lasts from the birth of the fetus until the placenta is delivered. This stage may be as short as 3 minutes or as long as 1 hour. The fourth stage of labor, recovery, lasts approximately 2 hours after the delivery of the placenta.

What is the correct placement of the tocotransducer for effective EFM? a. Over the uterine fundus b. On the fetal scalp c. Inside the uterus d. Over the mother's lower abdomen

A The tocotransducer monitors uterine activity and should be placed over the fundus, where the most intensive uterine contractions occur. The tocotransducer is for external use.

In assessing the immediate condition of the newborn after birth, a sample of cord blood may be a useful adjunct to the Apgar score. Cord blood is then tested for pH, carbon dioxide, oxygen, and base deficit or excess. Which clinical situation warrants this additional testing? (Select all that apply.) a. Low 5-minute Apgar score b. Intrauterine growth restriction (IUGR) c. Maternal thyroid disease d. Intrapartum fever e. Vacuum extraction

A, B, C, D The American College of Obstetricians and Gynecologists (ACOG) suggests obtaining cord blood values in all of these clinical situations except for vacuum extractions deliveries. Cord blood gases should also be performed for multifetal pregnancies or abnormal FHR tracings. Samples can be drawn from both the umbilical artery and the umbilical vein. Results may indicate that fetal compromise has occurred.

What are the complications and risks associated with cesarean births? (Select all that apply.) a. Pulmonary edema b. Wound dehiscence c. Hemorrhage d. Urinary tract infections e. Fetal injuries

A, B, C, D, E Pulmonary edema, wound dehiscence, hemorrhage, urinary tract infections, and fetal injuries are possible complications and risks associated with cesarean births.

If a woman is at risk for thrombus and is not ready to ambulate, which nursing intervention would the nurse use? (Select all that apply.) a. Putting her in antiembolic stockings (thromboembolic deterrent [TED] hose) and/or sequential compression device (SCD) boots b. Having her flex, extend, and rotate her feet, ankles, and legs c. Having her sit in a chair d. Immediately notifying the physician if a positive Homans sign occurs e. Promoting bed rest

A, B, D Sitting immobile in a chair does not help; bed exercise and prophylactic footwear might. TED hose and SCD boots are recommended. The client should be encouraged to ambulate with assistance, not remain in bed. Bed exercises are useful. A positive Homans sign (calf muscle pain or warmth, redness, tenderness) requires the physician's immediate attention.

Many new mothers experience some type of nipple pain during the first weeks of initiating breastfeeding. Should this pain be severe or persistent, it may discourage or inhibit breastfeeding altogether. Which factors might contribute to this pain? (Select all that apply.) a. Improper feeding position b. Large-for-gestational age infant c. Fair skin d. Progesterone deficiency e. Flat or retracted nipples

A, C, E Nipple lesions may manifest as chapped, cracked, bleeding, sore, erythematous, edematous, or blistered nipples. Factors that contribute to nipple pain include improper positioning or a failure to break suction before removing the baby from the breast. Flat or retracted nipples along with the use of nipple shields, breast shells, or plastic breast pads also contribute to nipple pain. Women with fair skin are more likely to develop sore and cracked nipples. Preventing nipple soreness is preferable to treating soreness after it appears. Vigorous feeding may be a contributing factor, which may be the case with any size infant, not just infants who are large for gestational age. Estrogen or dietary deficiencies can contribute to nipple soreness.

Which statement is the best rationale for assessing the maternal vital signs between uterine contractions? a. During a contraction, assessing the fetal heart rate is the priority. b. Maternal circulating blood volume temporarily increases during contractions. c. Maternal blood flow to the heart is reduced during contractions. d. Vital signs taken during contractions are not accurate.

B During uterine contractions, blood flow to the placenta temporarily stops, causing a relative increase in the mother's blood volume, which, in turn, temporarily increases blood pressure and slows the pulse. Monitoring fetal responses to the contractions is important; however, this question concerns the maternal vital signs. Maternal blood flow is increased during a contraction. Vital signs are altered by contractions but are considered accurate for that period.

Which description of the phases of the first stage of labor is most accurate? a. Latent: mild, regular contractions; no dilation; bloody show b. Active: moderate, regular contractions; 4 to 7 cm dilation c. Lull: no contractions; dilation stable d. Transition: very strong but irregular contractions; 8 to 10 cm dilation

B The active phase is characterized by moderate and regular contractions, 4 to 7 cm dilation, and duration of 3 to 6 hours. The latent phase is characterized by mild-to-moderate and irregular contractions, dilation up to 3 cm, brownish-to-pale pink mucus, and duration of 6 to 8 hours. No official "lull" phase exists in the first stage. The transition phase is characterized by strong to very strong and regular contractions, 8 to 10 cm dilation, and duration of 20 to 40 minutes.

During labor a fetus displays an average FHR of 135 beats per minute over a 10-minute period. Which statement best describes the status of this fetus? a. Bradycardia b. Normal baseline heart rate c. Tachycardia d. Hypoxia

B The baseline FHR is measured over 10 minutes; a normal range is 110 to 160 beats per minute. Bradycardia is a FHR less than 110 beats per minute for 10 minutes or longer. Tachycardia is a FHR higher than 160 beats per minutes for 10 minutes or longer. Hypoxia is an inadequate supply of oxygen; no indication of hypoxia exists with a baseline FHR in the normal range.

A laboring woman has received meperidine (Demerol) intravenously (IV), 90 minutes before giving birth. Which medication should be available to reduce the postnatal effects of meperidine on the neonate? a. Fentanyl (Sublimaze) b. Promethazine (Phenergan) c. Naloxone (Narcan) d. Nalbuphine (Nubain)

C An opioid antagonist can be given to the newborn as one part of the treatment for neonatal narcosis, which is a state of central nervous system (CNS) depression in the newborn produced by an opioid. Opioid antagonists, such as naloxone (Narcan), can promptly reverse the CNS depressant effects, especially respiratory depression. Fentanyl (Sublimaze), promethazine (Phenergan), and nalbuphine (Nubain) do not act as opioid antagonists to reduce the postnatal effects of meperidine on the neonate.

Nurses with an understanding of cultural differences regarding likely reactions to pain may be better able to help their clients. Which clients may initially appear very stoic but then become quite vocal as labor progresses until late in labor, when they become more vocal and request pain relief? a. Chinese b. Arab or Middle Eastern c. Hispanic d. African-American

C Hispanic women may be stoic early in labor but more vocal and ready for medications later. Chinese women may not show reactions to pain. Medical interventions must be offered more than once. Arab or Middle Eastern women may be vocal in response to labor pain from the start; they may prefer pain medications. African-American women may openly express pain; the use of medications for pain is more likely to vary with the individual.

A primigravida at 40 weeks of gestation is having uterine contractions every to 2 minutes and states that they are very painful. Her cervix is dilated 2 cm and has not changed in 3 hours. The woman is crying and wants an epidural. What is the likely status of this woman's labor? a. She is exhibiting hypotonic uterine dysfunction. b. She is experiencing a normal latent stage. c. She is exhibiting hypertonic uterine dysfunction. d. She is experiencing precipitous labor.

C The contraction pattern observed in this woman signifies hypertonic uterine activity. Typically, uterine activity in this phase occurs at 4- to 5-minute intervals lasting 30 to 45 seconds. Women who experience hypertonic uterine dysfunction, or primary dysfunctional labor, are often anxious first-time mothers who are having painful and frequent contractions that are ineffective at causing cervical dilation or effacement to progress. With hypotonic uterine dysfunction, the woman initially makes normal progress into the active stage of labor; then the contractions become weak and inefficient or stop altogether. Precipitous labor is one that lasts less than 3 hours from the onset of contractions until time of birth.

A labor and delivery nurse should be cognizant of which information regarding how the fetus moves through the birth canal? a. Fetal attitude describes the angle at which the fetus exits the uterus. b. Of the two primary fetal lies, the horizontal lie is that in which the long axis of the fetus is parallel to the long axis of the mother. c. Normal attitude of the fetus is called general flexion. d. Transverse lie is preferred for vaginal birth.

C The normal attitude of the fetus is called general flexion. The fetal attitude is the relationship of the fetal body parts to each one another. The horizontal lie is perpendicular to the mother; in the longitudinal (or vertical) lie, the long axes of the fetus and the mother are parallel. Vaginal birth cannot occur if the fetus stays in a transverse lie.

Which basic type of pelvis includes the correct description and percentage of occurrence in women? a. Gynecoid: classic female pelvis; heart shaped; 75% b. Android: resembling the male pelvis; wide oval; 15% c. Anthropoid: resembling the pelvis of the ape; narrow; 10% d. Platypelloid: flattened, wide, and shallow pelvis; 3%

D A platypelloid pelvis is flattened, wide, and shallow; approximately 3% of women have this shape. The gynecoid pelvis is the classic female shape, slightly ovoid and rounded; approximately 50% of women have this shape. An android or malelike pelvis is heart shaped; approximately 23% of women have this shape. An anthropoid or apelike pelvis is oval and wide; approximately 24% of women have this shape.

Certain changes stimulate chemoreceptors in the aorta and carotid bodies to prepare the fetus for initiating respirations immediately after birth. Which change in fetal physiologic activity is not part of this process? a. Fetal lung fluid is cleared from the air passages during labor and vaginal birth. b. Fetal partial pressure of oxygen (PO2) decreases. c. Fetal partial pressure of carbon dioxide in arterial blood (PaCO2) increases. d. Fetal respiratory movements increase during labor.

D Fetal respiratory movements actually decrease during labor. Fetal lung fluid is cleared from the air passages during labor and vaginal birth. Fetal PO2 decreases, and fetal PaCO2 increases.

What are the most common causes for subinvolution of the uterus? a. Postpartum hemorrhage and infection b. Multiple gestation and postpartum hemorrhage c. Uterine tetany and overproduction of oxytocin d. Retained placental fragments and infection

D Subinvolution is the failure of the uterus to return to a nonpregnant state. The most common causes of subinvolution are retained placental fragments and infection. Subinvolution may be caused by an infection and result in hemorrhage. Multiple gestations may cause uterine atony, resulting in postpartum hemorrhaging. Uterine tetany and overproduction of oxytocin do not cause subinvolution.

The Valsalva maneuver can be described as the process of making a forceful bearing-down attempt while holding one's breath with a closed glottis and a tightening of the abdominal muscles. When is it appropriate to instruct the client to use this maneuver? a. During the second stage to enhance the movement of the fetus b. During the third stage to help expel the placenta c. During the fourth stage to expel blood clots d. Not at all

D The client should not be instructed to use this maneuver. This process stimulates the parasympathetic division of the autonomic nervous system and produces a vagal response (decrease in heart rate and blood pressure.) An alternative method includes instructing the client to perform open-mouth and open-glottis breathing and pushing.

During which phase of maternal adjustment will the mother relinquish the baby of her fantasies and accept the real baby? a. Letting go b. Taking hold c. Taking in d. Taking on

A Accepting the real infant and relinquishing the fantasy infant occurs during the letting-go phase of maternal adjustment. During the taking-hold phase, the mother assumes responsibility for her own care and shifts her attention to the infant. In the taking-in phase, the mother is primarily focused on her own needs. A taking-on phase of maternal adjustment does not exist.

Which information regarding the procedures and criteria for admitting a woman to the hospital labor unit is important for the nurse to understand? a. Client is considered to be in active labor when she arrives at the facility with contractions. b. Client can have only her male partner or predesignated doula with her at assessment. c. Children are not allowed on the labor unit. d. Non-English speaking client must bring someone to translate.

A According to the Emergency Medical Treatment and Active Labor Act (EMTALA), a woman is entitled to active labor care and is presumed to be in true labor until a qualified health care provider certifies otherwise. A woman may have anyone she wishes present for her support. An interpreter must be provided by the hospital, either in person or by a telephonic service. Siblings of the new infant may be allowed at the delivery, depending on hospital policy and adequate preparation and supervision.

Nursing care measures are commonly offered to women in labor. Which nursing measure reflects the application of the gate-control theory? a. Massage the woman's back. b. Change the woman's position. c. Give the prescribed medication. d. Encourage the woman to rest between contractions.

A According to the gate-control theory, pain sensations travel along sensory nerve pathways to the brain, but only a limited number of sensations, or messages, can travel through these nerve pathways at one time. Distraction techniques, such as massage or stroking, music, focal points, and imagery, reduce or completely block the capacity of the nerve pathways to transmit pain. These distractions are thought to work by closing down a hypothetic gate in the spinal cord, thus preventing pain signals from reaching the brain. The perception of pain is thereby diminished. Changing the woman's position, administering pain medication, and resting between contractions do not reduce or block the capacity of the nerve pathways to transmit pain using the gate-control theory.

Which alteration in the FHR pattern would indicate the potential need for an amnioinfusion? a. Variable decelerations b. Late decelerations c. Fetal bradycardia d. Fetal tachycardia

A Amnioinfusion is used during labor to either dilute meconium-stained amniotic fluid or supplement the amount of amniotic fluid to reduce the severity of variable FHR decelerations caused by cord compression. Late decelerations are unresponsive to amnioinfusion. Amnioinfusion is not appropriate for the treatment of fetal bradycardia and has no bearing on fetal tachycardia.

A woman gave birth to a 7-pound, 6-ounce infant girl 1 hour ago. The birth was vaginal and the estimated blood loss (EBL) was 1500 ml. When evaluating the woman's vital signs, which finding would be of greatest concern to the nurse? a. Temperature 37.9° C, heart rate 120 beats per minute (bpm), respirations 20 breaths per minute, and blood pressure 90/50 mm Hg b. Temperature 37.4° C, heart rate 88 bpm, respirations 36 breaths per minute, and blood pressure 126/68 mm Hg c. Temperature 38° C, heart rate 80 bpm, respirations 16 breaths per minute, and blood pressure 110/80 mm Hg d. Temperature 36.8° C, heart rate 60 bpm, respirations 18 breaths per minute, and blood pressure 140/90 mm Hg

A An EBL of 1500 ml with tachycardia and hypotension suggests hypovolemia caused by excessive blood loss. Temperature 37.4° C, heart rate 88 bpm, respirations 36 breaths per minute, and blood pressure 126/68 mm Hg are normal vital signs except for an increased respiratory rate, which may be secondary to pain from the birth. Temperature 38° C, heart rate 80 bpm, respirations 16 breaths per minute, and blood pressure 110/80 mm Hg are normal vital signs except for the temperature, which may increase to 38° C during the first 24 hours as a result of the dehydrating effects of labor. Temperature 36.8° C, heart rate 60 bpm, respirations 18 breaths per minute, and blood pressure 140/90 mm Hg are normal vital signs, although the blood pressure is slightly elevated, which may be attributable to the use of oxytocic medications.

Rho immune globulin will be ordered postpartum if which situation occurs? a. Mother Rh-, baby Rh+ b. Mother Rh-, baby Rh- c. Mother Rh+, baby Rh+ d. Mother Rh+, baby Rh-

A An Rh- mother delivering an Rh+ baby may develop antibodies to fetal cells that entered her bloodstream when the placenta separated. The Rho immune globulin works to destroy the fetal cells in the maternal circulation before sensitization occurs. If mother and baby are both Rh+ or Rh- the blood types are alike, so no antibody formation would be anticipated. If the Rh+ blood of the mother comes in contact with the Rh- blood of the infant, no antibodies would develop because the antigens are in the mother's blood, not in the infant's.

A woman in preterm labor at 30 weeks of gestation receives two 12-mg intramuscular (IM) doses of betamethasone. What is the purpose of this pharmacologic intervention? a. To stimulate fetal surfactant production b. To reduce maternal and fetal tachycardia associated with ritodrine administration c. To suppress uterine contractions d. To maintain adequate maternal respiratory effort and ventilation during magnesium sulfate therapy

A Antenatal glucocorticoids administered as IM injections to the mother accelerate fetal lung maturity. Propranolol (Inderal) is given to reduce the effects of ritodrine administration. Betamethasone has no effect on uterine contractions. Calcium gluconate is given to reverse the respiratory depressive effects of magnesium sulfate therapy.

New parents express concern that because of the mother's emergency cesarean birth under general anesthesia, they did not have the opportunity to hold and bond with their daughter immediately after her birth. Which information should the nurse's response convey? a. Attachment, or bonding, is a process that occurs over time and does not require early contact. b. Time immediately after birth is a critical period for humans. c. Early contact is essential for optimal parent-infant relationships. d. These new parents should just be happy that the infant is healthy.

A Attachment occurs over time and does not require early contact. Although a delay in contact does not necessarily mean that attachment is inhibited, additional psychologic energy may be necessary to achieve the same effect. The formerly accepted definition of bonding held that the period immediately after birth was critical for bonding to occur. Research since has indicated that parent-infant attachment occurs over time. A delay does not inhibit the process. Parent-infant attachment involves activities such as touching, holding, and gazing; it is not exclusively eye contact. Telling the parents that they should be happy that the infant is healthy is inappropriate; it may be received as derogatory and belittling.

Which description most accurately describes the augmentation of labor? a. Is part of the active management of labor that is instituted when the labor process is unsatisfactory b. Relies on more invasive methods when oxytocin and amniotomy have failed c. Is a modern management term to cover up the negative connotations of forceps-assisted birth d. Uses vacuum cups

A Augmentation is part of the active management of labor that stimulates uterine contractions after labor has started but is not progressing satisfactorily. Augmentation uses amniotomy and oxytocin infusion, as well as some more gentle, noninvasive methods. Forceps-assisted births are less common than in the past and not considered a method of augmentation. A vacuum-assisted delivery occurs during childbirth if the mother is too exhausted to push. Vacuum extraction is not considered an augmentation methodology.

After delivery, excess hypertrophied tissue in the uterus undergoes a period of self-destruction. What is the correct term for this process? a. Autolysis b. Subinvolution c. Afterpains d. Diastasis

A Autolysis is caused by a decrease in hormone levels. Subinvolution is failure of the uterus to return to a nonpregnant state. Afterpains are caused by uterine cramps 2 to 3 days after birth. Diastasis refers to the separation of muscles.

When managing the care of a woman in the second stage of labor, the nurse uses various measures to enhance the progress of fetal descent. Which instruction best describes these measures? a. Encouraging the woman to try various upright positions, including squatting and standing b. Telling the woman to start pushing as soon as her cervix is fully dilated c. Continuing an epidural anesthetic so pain is reduced and the woman can relax d. Coaching the woman to use sustained, 10- to 15-second, closed-glottis bearing-down efforts with each contraction

A Both upright and squatting positions may enhance the progress of fetal descent. Many factors dictate when a woman should begin pushing. Complete cervical dilation is necessary, but complete dilation is only one factor. If the fetal head is still in a higher pelvic station, then the physician or midwife may allow the woman to "labor down" if the woman is able (allowing more time for fetal descent and thereby reducing the amount of pushing needed). The epidural may mask the sensations and muscle control needed for the woman to push effectively. Closed glottic breathing may trigger the Valsalva maneuver, which increases intrathoracic and cardiovascular pressures, reducing cardiac output and inhibiting perfusion of the uterus and placenta. In addition, holding her breath for longer than 5 to 7 seconds diminishes the perfusion of oxygen across the placenta and results in fetal hypoxia.

A client is experiencing back labor and complains of intense pain in her lower back. Which measure would best support this woman in labor? a. Counterpressure against the sacrum b. Pant-blow (breaths and puffs) breathing techniques c. Effleurage d. Conscious relaxation or guided imagery

A Counterpressure is steady pressure applied by a support person to the sacral area with the fist or heel of the hand. This technique helps the woman cope with the sensations of internal pressure and pain in the lower back. The pain management techniques of pant-blow, effleurage, and conscious relaxation or guided imagery are usually helpful for contractions per the gate-control theory.

When should discharge instruction, or the teaching plan that tells the woman what she needs to know to care for herself and her newborn, officially begin? a. At the time of admission to the nurse's unit b. When the infant is presented to the mother at birth c. During the first visit with the physician in the unit d. When the take-home information packet is given to the couple

A Discharge planning, the teaching of maternal and newborn care, begins on the woman's admission to the unit, continues throughout her stay, and actually never ends as long as she has contact with medical personnel.

The nurse expects which maternal cardiovascular finding during labor? a. Increased cardiac output b. Decreased pulse rate c. Decreased white blood cell (WBC) count d. Decreased blood pressure

A During each contraction, 400 ml of blood is emptied from the uterus into the maternal vascular system, which increases cardiac output by approximately 10% to 15% during the first stage of labor and by approximately 30% to 50% in the second stage of labor. The heart rate increases slightly during labor. The WBC count can increase during labor. During the first stage of labor, uterine contractions cause systolic readings to increase by approximately 10 mm Hg. During the second stage, contractions may cause systolic pressures to increase by 30 mm Hg and diastolic readings to increase by 25 mm Hg.

A, B, C, E Culture: A woman's sociocultural roots influence how she perceives, interprets, and responds to pain during childbirth. Some cultures encourage loud and vigorous expressions of pain, whereas others value self-control. The nurse should avoid praising some behaviors (stoicism) while belittling others (noisy expression). Anxiety and fear: Extreme anxiety and fear magnify the sensitivity to pain and impair a woman's ability to tolerate it. Anxiety and fear increase muscle tension in the pelvic area, which counters the expulsive forces of uterine contractions and pushing efforts. Previous experiences with pain: Fear and withdrawal are a natural response to pain during labor. Learning about these normal sensations ahead of time helps a woman suppress her natural reactions of fear regarding the impending birth. If a woman previously had a long and difficult labor, she is likely to be anxious. She may also have learned ways to cope and may use these skills to adapt to the present labor experience. Support systems: An anxious partner is less able to provide help and support to a woman during labor. A woman's family and friends can be an important source of support if they convey realistic and positive information about labor and delivery. Although the intervention of caregivers may be necessary for the well-being of the woman and her fetus, some interventions add discomfort to the natural pain of labor (i.e., fetal monitor straps, IV lines).

A Early decelerations are the fetus' response to fetal head compression; these are considered benign, and interventions are not necessary. Variable decelerations are associated with umbilical cord compression. Late decelerations are associated with uteroplacental insufficiency. Spontaneous rupture of membranes has no bearing on the FHR unless the umbilical cord prolapses, which would result in variable or prolonged bradycardia.

Which clinical finding or intervention might be considered the rationale for fetal tachycardia to occur? a. Maternal fever b. Umbilical cord prolapse c. Regional anesthesia d. Magnesium sulfate administration

A Fetal tachycardia can be considered an early sign of fetal hypoxemia and may also result from maternal or fetal infection. Umbilical cord prolapse, regional anesthesia, and the administration of magnesium sulfate will each more likely result in fetal bradycardia, not tachycardia.

Which client would not be a suitable candidate for internal EFM? a. Client who still has intact membranes b. Woman whose fetus is well engaged in the pelvis c. Pregnant woman who has a comorbidity of obesity d. Client whose cervix is dilated to 4 to 5 cm

A For internal EFM, the membranes must have ruptured and the cervix must be dilated at least 2 to 3 cm. The presenting part must be low enough to allow placement of the spiral electrode necessary for internal EFM. The accuracy of EFM is not affected by maternal size. However, evaluating fetal well-being using external EFM may be more difficult on an obese client. The client whose cervix is dilated to 4 to 5 cm is indeed a candidate for internal monitoring.

Which description of the four stages of labor is correct for both the definition and the duration? a. First stage: onset of regular uterine contractions to full dilation; less than 1 hour to 20 hours b. Second stage: full effacement to 4 to 5 cm; visible presenting part; 1 to 2 hours c. Third stage: active pushing to birth; 20 minutes (multiparous woman), 50 minutes (nulliparous woman) d. Fourth stage: delivery of the placenta to recovery; 30 minutes to 1 hour

A Full dilation may occur in less than 1 hour, but in first-time pregnancies full dilation can take up to 20 hours. The second stage of labor extends from full dilation to birth and takes an average of 20 to 50 minutes, although 2 hours is still considered normal. The third stage of labor extends from birth to the expulsion of the placenta and usually takes a few minutes. The fourth stage begins after the expulsion of the placenta and lasts until homeostasis is reestablished (approximately 2 hours).

As the United States and Canada continue to become more culturally diverse, recognizing a wide range of varying cultural beliefs and practices is increasingly important for the nursing staff. A client is from which country if she requests to have the baby's father in attendance? a. Mexico b. China c. Iran d. India

A Hispanic women routinely have fathers and female relatives in attendance during the second stage of labor. The father of the baby is expected to provide encouragement, support, and reassurance that all will be well. In China, fathers are usually not present. The side-lying position is preferred for labor and birth because it is believed that this will reduce trauma to the infant. In China, the client has a stoic response to pain. In Iran, the father will not be present. Female support persons and female health care providers are preferred. For many, a male caregiver is unacceptable. In India, the father is usually not present, but female relatives are usually in attendance. Natural childbirth methods are preferred.

In recovery, if a woman is asked to either raise her legs (knees extended) off the bed or flex her knees, and then place her feet flat on the bed and raise her buttocks well off the bed, the purpose of this exercise is to assess what? a. Recovery from epidural or spinal anesthesia b. Hidden bleeding underneath her c. Flexibility d. Whether the woman is a candidate to go home after 6 hour

A If the numb or prickly sensations are gone from her legs after these movements, then she has likely recovered from the epidural or spinal anesthesia. Assessing the client for bleeding beneath her buttocks before discharge from the recovery is always important; however, she should be rolled to her side for this assessment. The nurse is not required to assess the woman for flexibility. This assessment is performed to evaluate whether the client has recovered from spinal anesthesia, not to determine if she is a candidate for early discharge.

Which presentation is accurately described in terms of both the resenting part and the frequency of occurrence? a. Cephalic: occiput, at least 96% b. Breech: sacrum, 10% to 15% c. Shoulder: scapula, 10% to 15% d. Cephalic: cranial, 80% to 85%

A In cephalic presentations (head first), the presenting part is the occiput; this presentation occurs in 96% of births. In a breech birth, the sacrum emerges first; this presentation occurs in approximately 3% of births. In shoulder presentations, the scapula emerges first; this presentation occurs in only 1% of births. In a cephalic presentation, the part of the head or cranium that emerges first is the occiput; cephalic presentations occur in 96% of births.

In many hospitals, new mothers are routinely presented with gift bags containing samples of infant formula. This practice is inconsistent with what? a. Baby Friendly Hospital Initiative b. Promotion of longer periods of breastfeeding c. Perception of being supportive to both bottle feeding and breastfeeding mothers d. Association with earlier cessation of breastfeeding

A Infant formula should not be given to mothers who are breastfeeding. Such gifts are associated with early cessation of breastfeeding. Baby Friendly USA prohibits the distribution of any gift bags or formula to new mothers.

Which statement by the nurse can assist a new father in his transition to parenthood? a. Pointing out that the infant turned at the sound of his voice b. Encouraging him to go home to get some sleep c. Telling him to tape the infant's diaper a different way d. Suggesting that he let the infant sleep in the bassinet

A Infants respond to the sound of voices. Because attachment involves a reciprocal interchange, observing the interaction between parent and infant is very important. Separation of the parent and infant does not encourage parent-infant attachment. Educating the parent in infant care techniques is important, but the manner in which a diaper is taped is not relevant and does not enhance parent-infant interactions. Parent-infant attachment involves touching, holding, and cuddling. It is appropriate for a father to want to hold the infant as the baby sleeps.

The exact cause of preterm labor is unknown but believed to be multifactorial. Infection is thought to be a major factor in many preterm labors. Which type of infection has not been linked to preterm birth? a. Viral b. Periodontal c. Cervical d. Urinary tract

A Infections that increase the risk of preterm labor and birth are bacterial and include cervical, urinary tract, periodontal, and other bacterial infections. Therefore, early, continual, and comprehensive participation by the client in her prenatal care is important. Recent evidence has shown a link between periodontal infections and preterm labor. Researchers recommend regular dental care before and during pregnancy, oral assessment as a routine part of prenatal care, and scrupulous oral hygiene to prevent periodontal infections.

Which nursing intervention should be immediately performed after the forceps-assisted birth of an infant? a. Assessing the infant for signs of trauma b. Administering prophylactic antibiotic agents to the infant c. Applying a cold pack to the infant's scalp d. Measuring the circumference of the infant's head

A The infant should be assessed for bruising or abrasions at the site of application, facial palsy, and subdural hematoma. Prophylactic antibiotics are not necessary with a forceps delivery. A cold pack would place the infant at risk for cold stress and is contraindicated. Measuring the circumference of the head is part of the initial nursing assessment.

While evaluating an external monitor tracing of a woman in active labor, the nurse notes that the FHR for five sequential contractions begins to decelerate late in the contraction, with the nadir of the decelerations occurring after the peak of the contraction. What is the nurse's first priority? a. Change the woman's position. b. Notify the health care provider. c. Assist with amnioinfusion d. Insert a scalp electrode.

A Late FHR decelerations may be caused by maternal supine hypotension syndrome. These decelerations are usually corrected when the woman turns onto her side to displace the weight of the gravid uterus from the vena cava. If the fetus does not respond to primary nursing interventions for late decelerations, then the nurse should continue with subsequent intrauterine resuscitation measures and notify the health care provider. An amnioinfusion may be used to relieve pressure on an umbilical cord that has not prolapsed. The FHR pattern associated with this situation most likely will reveal variable decelerations. Although a fetal scalp electrode will provide accurate data for evaluating the well-being of the fetus, it is not a nursing intervention that will alleviate late decelerations nor is it the nurse's first priority.

Which component of the physical examination are Leopold's maneuvers unable to determine? a. Gender of the fetus b. Number of fetuses c. Fetal lie and attitude d. Degree of the presenting part's descent into the pelvis

A Leopold's maneuvers help identify the number of fetuses, the fetal lie and attitude, and the degree of descent of the presenting part into the pelvis. The gender of the fetus cannot be determined by performing Leopold's maneuvers.

Which nursing intervention would result in an increase in maternal cardiac output? a. Change in position b. Oxytocin administration c. Regional anesthesia d. IV analgesic

A Maternal supine hypotension syndrome is caused by the weight and pressure of the gravid uterus on the ascending vena cava when the woman is in a supine position. This position reduces venous return to the woman's heart, as well as cardiac output, and subsequently reduces her blood pressure. The nurse can encourage the woman to change positions and to avoid the supine position. Oxytocin administration, regional anesthesia, and IV analgesic may reduce maternal cardiac output.

When the infant's behaviors and characteristics call forth a corresponding set of maternal behaviors and characteristics, what is the correct term for this behavior? a. Mutuality b. Bonding c. Claiming d. Acquaintance

A Mutuality extends the concept of attachment and includes a shared set of behaviors as part of the bonding process. Bonding is the process during which parents form an emotional attachment to their infant over time. Claiming is the process during which parents identify their new baby in terms of the infant's likeness to other family members and their differences and uniqueness. Similar to mutuality, acquaintance is part of attachment. It describes how parents get to know their baby during the immediate postpartum period through eye contact, touching, and talking.

What are the legal responsibilities of the perinatal nurses? a. Correctly interpreting FHR patterns, initiating appropriate nursing interventions, and documenting the outcomes b. Greeting the client on arrival, assessing her status, and starting an IV line c. Applying the external fetal monitor and notifying the health care provider d. Ensuring that the woman is comfortable

A Nurses who care for women during childbirth are legally responsible for correctly interpreting FHR patterns, initiating appropriate nursing interventions based on those patterns, and documenting the outcomes of those interventions. Greeting the client on arrival, assessing her, and starting an IV line are activities that should be performed when any client arrives to the maternity unit. The nurse is not the only one legally responsible for performing these functions. Applying the external fetal monitor and notifying the health care provider is a nursing function that is part of the standard of care for all obstetric clients and falls within the registered nurse's scope of practice. Everyone caring for the pregnant woman should ensure that both she and her support partner are comfortable.

Which statement best describes a normal uterine activity pattern in labor? a. Contractions every 2 to 5 minutes b. Contractions lasting approximately 2 minutes c. Contractions approximately 1 minute apart d. Contraction intensity of approximately 500 mm Hg with relaxation at 50 mm Hg

A Overall contraction frequency generally ranges from two to five contractions per 10 minutes of labor, with lower frequencies during the first stage and higher frequencies observed during the second stage. Contraction duration remains fairly stable throughout the first and second stages, ranging from 45 to 80 seconds, generally not exceeding 90 seconds. Contractions 1 minute apart are occurring too often and would be considered an abnormal labor pattern. The intensity of uterine contractions generally ranges from 25 to 50 mm Hg in the first stage of labor and may rise to more than 80 mm Hg in the second stage.

When assessing a multiparous woman who has just given birth to an 8-pound boy, the nurse notes that the woman's fundus is firm and has become globular in shape. A gush of dark red blood comes from her vagina. What is the nurse's assessment of the situation? a. The placenta has separated. b. A cervical tear occurred during the birth. c. The woman is beginning to hemorrhage. d. Clots have formed in the upper uterine segment.

A Placental separation is indicated by a firmly contracting uterus, a change in the uterus from a discoid to a globular ovoid shape, a sudden gush of dark red blood from the introitus, an apparent lengthening of the umbilical cord, and a finding of vaginal fullness. Cervical tears that do not extend to the vagina result in minimal blood loss. Signs of hemorrhage are a boggy uterus, bright red vaginal bleeding, alterations in vital signs, pallor, lightheadedness, restlessness, decreased urinary output, and alteration in the level of consciousness. If clots have formed in the upper uterine segment, then the nurse would expect to find the uterus boggy and displaced to the side.

The nurse should be cognizant of which physiologic effect of pain? a. Predominant pain of the first stage of labor is visceral pain that is located in the lower portion of the abdomen. b. Referred pain is the extreme discomfort experienced between contractions. c. Somatic pain of the second stage of labor is more generalized and related to fatigue. d. Pain during the third stage is a somewhat milder version of the pain experienced during the second stage.

A Predominant pain comes from cervical changes, the distention of the lower uterine segment, and uterine ischemia. Referred pain occurs when the pain that originates in the uterus radiates to the abdominal wall, lumbosacral area of the back, iliac crests, and gluteal area. Second-stage labor pain is intense, sharp, burning, and localized. Third-stage labor pain is similar to that of the first stage.

A hospital has a number of different perineal pads available for use. A nurse is observed soaking several of them and writing down what she sees. What goal is the nurse attempting to achieve by performing this practice? a. To improve the accuracy of blood loss estimation, which usually is a subjective assessment b. To determine which pad is best c. To demonstrate that other nurses usually underestimate blood loss d. To reveal to the nurse supervisor that one of them needs some time off

A Saturation of perineal pads is a critical indicator of excessive blood loss; anything done to help in the assessment is valuable. The nurse is noting the saturation volumes and soaking appearances. Instead of determining which pad is best, the nurse is more likely noting saturation volumes and soaking appearances to improve the accuracy of estimated blood loss. Nurses usually overestimate blood loss. Soaking perineal pads and writing down the results does not indicate the need for time off of work.

Which alterations in the perception of pain by a laboring client should the nurse understand? a. Sensory pain for nulliparous women is often greater than for multiparous women during early labor. b. Affective pain for nulliparous women is usually less than for multiparous women throughout the first stage of labor. c. Women with a history of substance abuse experience more pain during labor. d. Multiparous women have more fatigue from labor and therefore experience more pain.

A Sensory pain is greater for nulliparous women because their reproductive tract structures are less supple. Affective pain is greater for nulliparous women during the first stage but decreases for both nulliparous and multiparous during the second stage. Women with a history of substance abuse experience the same amount of pain as those without such a history. Nulliparous women have longer labors and therefore experience more fatigue.

Breathing patterns are taught to laboring women. Which breathing pattern should the nurse support for the woman and her coach during the latent phase of the first stage of labor if the couple has attended childbirth preparation classes? a. Slow-paced breathing b. Deep abdominal breathing c. Modified-paced breathing d. Patterned-paced breathing

A Slow-paced breathing is approximately one half the woman's normal breathing rate and is used during the early stages of labor when a woman can no longer walk or talk through her contractions. No such pattern called deep abdominal breathing exists in childbirth preparation. Modified-paced breathing is shallow breathing that is twice the woman's normal breathing rate. It is used when labor progresses and the woman can no longer maintain relaxation through paced breathing. Patterned-pace breathing is a fast, 4:1 breathe, breathe, breathe, blow pattern that is used during the transitional phase of labor just before pushing and delivery.

Which technique is an adequate means of controlling the birth of the fetal head during delivery in a vertex presentation? a. Ritgen maneuver b. Fundal pressure c. Lithotomy position d. De Lee apparatus

A The Ritgen maneuver extends the head during the actual birth and protects the perineum. Gentle, steady pressure against the fundus of the uterus facilitates vaginal birth. The lithotomy position has been commonly used in Western cultures, partly because it is convenient for the health care provider. The De Lee apparatus is used to suction fluid from the infant's mouth.

What is the most critical nursing action in caring for the newborn immediately after the birth? a. Keeping the airway clear b. Fostering parent-newborn attachment c. Drying the newborn and wrapping the infant in a blanket d. Administering eye drops and vitamin K

A The care given immediately after the birth focuses on assessing and stabilizing the newborn. Although fostering parent-newborn attachment is an important task for the nurse, it is not the most critical nursing action in caring for the newborn immediately after birth. The care given immediately after birth focuses on assessing and stabilizing the newborn. The nursing activities are (in order of importance) to maintain a patent airway, to support respiratory effort, and to prevent cold stress by drying the newborn and covering him or her with a warmed blanket or placing the newborn under a radiant warmer. After the newborn has been stabilized, the nurse assesses the newborn's physical condition, weighs and measures the newborn, administers prophylactic eye ointment and a vitamin K injection, affixes an identification bracelet, wraps the newborn in warm blankets, and then gives the newborn to the partner or to the mother of the infant

A primiparous woman is to be discharged from the hospital the following day with her infant girl. Which behavior indicates a need for further intervention by the nurse before the woman can be discharged? a. The woman is disinterested in learning about infant care. b. The woman continues to hold and cuddle her infant after she has fed her. c. The woman reads a magazine while her infant sleeps. d. The woman changes her infant's diaper and then shows the nurse the contents of the diaper.

A The client should be excited, happy, and interested or involved in infant care. A woman who is sad, tearful, or disinterested in caring for her infant may be exhibiting signs of depression or postpartum blues and may require further intervention. Holding and cuddling her infant after feeding is an appropriate parent-infant interaction. Taking time for herself while the infant is sleeping is an appropriate maternal action. Showing the nurse the contents of the diaper is appropriate because the mother is seeking approval from the nurse and notifying the nurse of the infant's elimination patterns.

A woman gave birth to an infant boy 10 hours ago. Where does the nurse expect to locate this woman's fundus? a. 1 centimeter above the umbilicus b. 2 centimeters below the umbilicus c. Midway between the umbilicus and the symphysis pubis d. Nonpalpable abdominally

A The fundus descends approximately 1 to 2 cm every 24 hours. Within 12 hours after delivery the fundus may be approximately 1 cm above the umbilicus. By the sixth postpartum week the fundus is normally halfway between the symphysis pubis and the umbilicus. The fundus should be easily palpated using the maternal umbilicus as a reference point.

Under which circumstance should the nurse assist the laboring woman into a hands-and-knees position? a. Occiput of the fetus is in a posterior position. b. Fetus is at or above the ischial spines. c. Fetus is in a vertex presentation. d. Membranes have ruptured.

A The hands-and-knees position is effective in helping to rotate the fetus from a posterior to an anterior position. Many women experience the irresistible urge to push when the fetus is at the level of the ischial spines. In some cases, this urge is felt before the woman is fully dilated. The woman should be instructed not to push until complete cervical dilation has occurred. No one position is correct for childbirth. The two most common positions assumed by women are the sitting and side-lying positions. The woman may be encouraged into a hands-and-knees position if the umbilical cord prolapsed when the membranes ruptured.

Several delivery changes in the integumentary system that appear during pregnancy disappear after birth, although not always completely. What change is almost certain to be completely reversed? a. Nail brittleness b. Darker pigmentation of the areolae and linea nigra c. Striae gravidarum on the breasts, abdomen, and thighs d. Spider nevi

A The nails return to their prepregnancy consistency and strength. Some women have permanent darker pigmentation of the areolae and linea nigra. Striae gravidarum (stretch marks) usually do not completely disappear. For some women, spider nevi persist indefinitely.

A nurse notes that an Eskimo woman does not cuddle or interact with her newborn other than to feed him, change his diapers or soiled clothes, and put him to bed. While evaluating this client's behavior with her infant, what realization does the nurse make? a. What appears to be a lack of interest in the newborn is, in fact, the cultural way of demonstrating intense love by attempting to ward off evil spirits. b. The woman is inexperienced in caring for a newborn. c. The woman needs a referral to a social worker for further evaluation of her parenting behaviors once she goes home with the newborn. d. Extra time needs to be planned for assisting the woman in bonding with her newborn.

A The nurse may observe an Eskimo mother who gives minimal care to her infant and refuses to cuddle or interact with her infant. The apparent lack of interest in the newborn is this cultural group's attempt to ward off evil spirits and actually reflects an intense love and concern for the infant. Inexperience in caring for newborns is not an issue. Cultural beliefs are important determinates of parenting behaviors. The woman's "lack of interest" is an Eskimo cultural behavior. Referring the woman to a social worker is not necessary in this situation. The lack of infant interaction is not a form of infant neglect; rather, it is a demonstration of love and concern for the infant. The nurse may observe the woman and may be concerned by the apparent lack of interest in the newborn when in fact her behavior is a cultural display of love and concern for the infant. Teaching the woman infant care is important, but acknowledging her cultural beliefs and practices is equally important.

What is the nurse's understanding of the appropriate role of primary and secondary powers? a. Primary powers are responsible for the effacement and dilation of the cervix. b. Effacement is generally well ahead of dilation in women giving birth for the first time; they are closer together in subsequent pregnancies. c. Scarring of the cervix caused by a previous infection or surgery may make the delivery a bit more painful, but it should not slow or inhibit dilation. d. Pushing in the second stage of labor is more effective if the woman can breathe deeply and control some of her involuntary needs to push, as the nurse directs.

A The primary powers are responsible for dilation and effacement; secondary powers are concerned with expulsion of the fetus. Effacement is generally well ahead of dilation in first-time pregnancies; they are closer together in subsequent pregnancies. Scarring of the cervix may slow dilation. Pushing is more effective and less fatiguing when the woman begins to push only after she has the urge to do so.

A pregnant woman's amniotic membranes have ruptured. A prolapsed umbilical cord is suspected. What intervention would be the nurse's highest priority? a. Placing the woman in the knee-chest position b. Covering the cord in sterile gauze soaked in saline c. Preparing the woman for a cesarean birth d. Starting oxygen by face mask

A The woman is assisted into a modified Sims position, Trendelenburg position, or the knee-chest position in which gravity keeps the pressure of the presenting part off the cord. Although covering the cord in sterile gauze soaked saline, preparing the woman for a cesarean, and starting oxygen by face mark are appropriate nursing interventions in the event of a prolapsed cord, the intervention of top priority would be positioning the mother to relieve cord compression.

Part of the nurse's role is assisting with pushing and positioning. Which guidance should the nurse provide to her client in active labor? a. Encourage the woman's cooperation in avoiding the supine position. b. Advise the woman to avoid the semi-Fowler position. c. Encourage the woman to hold her breath and tighten her abdominal muscles to produce a vaginal response. d. Instruct the woman to open her mouth and close her glottis, letting air escape after the push.

A The woman should maintain a side-lying position. The semi-Fowler position is the recommended side-lying position with a lateral tilt to the uterus. Encouraging the woman to hold her breath and tighten her abdominal muscles is the Valsalva maneuver, which should be avoided. Both the mouth and glottis should be open, allowing air to escape during the push.

A laboring woman is reclining in the supine position. What is the most appropriate nursing action at this time? a. Ask her to turn to one side. b. Elevate her feet and legs. c. Take her blood pressure. d. Determine whether fetal tachycardia is present.

A The woman's supine position may cause the heavy uterus to compress her inferior vena cava, thus reducing blood return to her heart and reducing placental blood flow. Elevating her legs will not relieve the pressure from the inferior vena cava. If the woman is allowed to stay in the supine position and blood flow to the placental is reduced significantly, then fetal tachycardia may occur. The most appropriate nursing action is to prevent this from occurring by turning the woman to her side. Blood pressure readings may be obtained when the client is in the appropriate and safest position.

The client has delivered by urgent caesarean birth for fetal compromise. Umbilical cord gases were obtained for acid-base determination. The pH is 6.9, partial pressure of carbon dioxide (PCO2) is elevated, and the base deficit is 11 mmol/L. What type of acidemia is displayed by the infant? a. Respiratory b. Metabolic c. Mixed d. Turbulent

A These findings are evidence of respiratory acidemia. Metabolic acidemia is expressed by a pH <7.20, normal carbon dioxide pressure, and a base excess of 12 mmol/L. Mixed acidemia is evidenced by a pH <7.20, elevated carbon dioxide pressure, and a base excess of 12 mmol/L. There is no such finding as turbulent acidemia.

A nurse caring for a woman in labor should understand that absent or minimal variability is classified as either abnormal or indeterminate. Which condition related to decreased variability is considered benign? a. Periodic fetal sleep state b. Extreme prematurity c. Fetal hypoxemia d. Preexisting neurologic injury

A When the fetus is temporarily in a sleep state, minimal variability is present. Periodic fetal sleep states usually last no longer than 30 minutes. A woman in labor with extreme prematurity may display a FHR pattern of minimal or absent variability. Abnormal variability may also be related to fetal hypoxemia and metabolic acidemia. Congenital anomalies or a preexisting neurologic injury may also result in absent or minimal variability. Other possible causes might be central nervous system (CNS) depressant medications, narcotics, or general anesthesia.

The nurse is caring for a client in early labor. Membranes ruptured approximately 2 hours earlier. This client is at increased risk for which complication? a. Intrauterine infection b. Hemorrhage c. Precipitous labor d. Supine hypotension

A When the membranes rupture, microorganisms from the vagina can ascend into the amniotic sac, causing chorioamnionitis and placentitis. ROM is not associated with fetal or maternal bleeding. Although ROM may increase the intensity of the contractions and facilitate active labor, it does not result in precipitous labor. ROM has no correlation with supine hypotension.

Conscious relaxation is associated with which method of childbirth preparation? a. Grantly Dick-Read childbirth method b. Lamaze method c. Bradley method d. Psychoprophylactic method

A With the Grantly Dick-Read method, women are taught to consciously and progressively relax different muscle groups throughout the body until a high degree of skill at relaxation is achieved. The Lamaze method combines controlled muscular relaxation with breathing techniques. The Bradley method advocates natural labor, without any form of anesthesia or analgesia, assisted by a husband-coach and using breathing techniques for labor. The psychoprophylactic method is another name for the Lamaze method.

The nurse should be aware of which information related to a woman's intake and output during labor? a. Traditionally, restricting the laboring woman to clear liquids and ice chips is being challenged because regional anesthesia is used more often than general anesthesia. b. Intravenous (IV) fluids are usually necessary to ensure that the laboring woman stays hydrated. c. Routine use of an enema empties the rectum and is very helpful for producing a clean, clear delivery. d. When a nulliparous woman experiences the urge to defecate, it often means birth will quickly follow.

A Women are awake with regional anesthesia and are able to protect their own airway, which reduces the worry over aspiration. Routine IV fluids during labor are unlikely to be beneficial and may be harmful. The routine use of an enema is, at best, ineffective and may be harmful. Having the urge to defecate followed by the birth of her fetus is true for a multiparous woman but not for a nulliparous woman.

Women who have participated in childbirth education classes often bring a birth plan with them to the hospital. Which items might this plan include? (Select all that apply.) a. Presence of companions b. Clothing to be worn c. Care and handling of the newborn d. Medical interventions e. Date of delivery

A, B, C, D The presence of companions, clothing to be worn, care and handling of the newborn, medical interventions, and environmental modifications all might be included in the couple's birth plan. Other items include the presence of nonessential medical personnel (students), labor activities such as the tub or ambulation, preferred comfort and relaxation methods, and any cultural or religious requirements. The expected date of delivery would not be part of a birth plan unless the client is scheduled for an elective cesarean birth.

Women who are obese are at risk for several complications during pregnancy and birth. Which of these would the nurse anticipate with an obese client? (Select all that apply.) a. Thromboembolism b. Cesarean birth c. Wound infection d. Breech presentation e. Hypertension

A, B, C, E A breech presentation is not a complication of pregnancy or birth for the client who is obese. Venous thromboembolism is a known risk for obese women. Therefore, the use of thromboembolism-deterrent (TED) hose and sequential compression devices may help decrease the chance for clot formation. Women should also be encouraged to ambulate as soon as possible. In addition to having an increased risk for complications with a cesarean birth, in general, obese women are also more likely to require an emergency cesarean birth. Many obese women have a pannus (i.e., large roll of abdominal fat) that overlies a lower transverse incision made just above the pubic area. The pannus causes the area to remain moist, which encourages the development of infection. Obese women are more likely to begin pregnancy with comorbidities such as hypertension and type 2 diabetes.

Emergency conditions during labor that would require immediate nursing intervention can arise with startling speed. Which situations are examples of such an emergency? (Select all that apply.) a. Nonreassuring or abnormal FHR pattern b. Inadequate uterine relaxation c. Vaginal bleeding d. Prolonged second stage e. Prolapse of the cord

A, B, C, E A nonreassuring or abnormal FHR pattern, inadequate uterine relaxation, vaginal bleeding, infection, and cord prolapse all constitute an emergency during labor that requires immediate nursing intervention. A prolonged second stage of labor after the upper limits for duration is reached. This is 3 hours for nulliparous women and 2 hours for multiparous women.

Which alternative approaches to relaxation have proven successful when working with the client in labor? (Select all that apply.) a. Aromatherapy b. Massage c. Hypnosis d. Cesarean birth e. Biofeedback

A, B, C, E Approaches to relaxation can include neuromuscular relaxation, aromatherapy, music, massage, imagery, hypnosis, or touch relaxation. Cesarean birth is a method of delivery, not a method of relaxation.

While developing an intrapartum care plan for the client in early labor, which psychosocial factors would the nurse recognize upon the client's pain experience? (Select all that apply.) a. Culture b. Anxiety and fear c. Previous experiences with pain d. Intervention of caregivers e. Support systems

A, B, C, E Culture: A woman's sociocultural roots influence how she perceives, interprets, and responds to pain during childbirth. Some cultures encourage loud and vigorous expressions of pain, whereas others value self-control. The nurse should avoid praising some behaviors (stoicism) while belittling others (noisy expression). Anxiety and fear: Extreme anxiety and fear magnify the sensitivity to pain and impair a woman's ability to tolerate it. Anxiety and fear increase muscle tension in the pelvic area, which counters the expulsive forces of uterine contractions and pushing efforts. Previous experiences with pain: Fear and withdrawal are a natural response to pain during labor. Learning about these normal sensations ahead of time helps a woman suppress her natural reactions of fear regarding the impending birth. If a woman previously had a long and difficult labor, she is likely to be anxious. She may also have learned ways to cope and may use these skills to adapt to the present labor experience. Support systems: An anxious partner is less able to provide help and support to a woman during labor. A woman's family and friends can be an important source of support if they convey realistic and positive information about labor and delivery. Although the intervention of caregivers may be necessary for the well-being of the woman and her fetus, some interventions add discomfort to the natural pain of labor (i.e., fetal monitor straps, IV lines).

At least five factors affect the process of labor and birth. These are easily remembered as the five Ps. Which factors are included in this process? (Select all that apply.) a. Passenger b. Passageway c. Powers d. Pressure e. Psychologic response

A, B, C, E The five Ps are passenger (fetus and placenta), passageway (birth canal), powers (contractions), position of the mother, and psychologic response. Pressure is not one of the five Ps.

Which societal factors have a strong influence on parental response to their infant? (Select all that apply.) a. An adolescent mother's egocentricity and unmet developmental needs interfere with her ability to parent effectively. b. An adolescent mother is likely to use less verbal instruction, be less responsive, and interact less positively than other mothers. c. Adolescent mothers have a higher documented incidence of child abuse. d. Mothers older than 35 years of age often deal with more stress related to work and career issues, as well as decreasing libido. e. Relationships between adolescent mothers and fathers are more stable than older adults.

A, B, D Adolescent mothers are more inclined to have a number of parenting difficulties that can benefit from counseling, but a higher incidence of child abuse is not one of them. As adolescent mothers move through the transition to parenthood, they can feel different from their peers, excluded from fun activities, and prematurely forced to enter the adult role. The conflict between their own desires and the infant's demands further contribute to the normal psychosocial stress of childbirth and parenting. Adolescent mothers provide warm and attentive physical care; however, they use less verbal interaction than older parents, and adolescents tend to be less responsive and to interact less positively with their infants than older mothers. Midlife mothers have many competencies; however, they are more likely to have to deal with career and sexual issues than are younger mothers. Relationships between adolescent parents tend to be less stable than among adults.

Because of its size and rigidity, the fetal head has a major effect on the birth process. Which bones comprise the structure of the fetal skull? (Select all that apply.) a. Parietal b. Temporal c. Fontanel d. Occipital e. Femoral

A, B, D The fetal skull has two parietal bones, two temporal bones, an occipital bone, and a frontal bone. The fontanels are membrane-filled spaces.

A woman has requested an epidural block for her pain. She is 5 cm dilated and 100% effaced. The baby is in a vertex position and is engaged. The nurse increases the woman's IV fluid for a preprocedural bolus. Before the initiation of the epidural, the woman should be informed regarding the disadvantages of an epidural block. Which concerns should the nurse share with this client? (Select all that apply.) a. Ability to move freely is limited. b. Orthostatic hypotension and dizziness may occur. c. Gastric emptying is not delayed. d. Higher body temperature may occur. e. Blood loss is not excessive.

A, B, D The woman's ability to move freely and to maintain control of her labor is limited, related to the use of numerous medical interventions (IV lines and electronic fetal monitoring [EFM]). Significant disadvantages of an epidural block include the occurrence of orthostatic hypotension, dizziness, sedation, and leg weakness. Women who receive an epidural block have a higher body temperature (38° C or higher), especially when labor lasts longer than 12 hours, and may result in an unnecessary neonatal workup for sepsis. An advantage of an epidural block is that blood loss is not excessive. Other advantages include the following: the woman remains alert and able to participate, good relaxation is achieved, airway reflexes remain intact, and only partial motor paralysis develops.

Which physiologic factors are reliable indicators of impending shock from postpartum hemorrhage? (Select all that apply.) a. Respirations b. Skin condition c. Blood pressure d. Level of consciousness e. Urinary output

A, B, D, E Blood pressure is not a reliable indicator; several more sensitive signs are available. Blood pressure does not drop until 30% to 40% of blood volume is lost. Respirations, pulse, skin condition, urinary output, and level of consciousness are more sensitive means of identifying hypovolemic shock.

Which concerns regarding parenthood are often expressed by visually impaired mothers? (Select all that apply.) a. Infant safety b. Transportation c. Ability to care for the infant d. Visually missing out e. Needing extra time for parenting activities to accommodate the visual limitations

A, B, D, E Concerns expressed by visually impaired mothers include infant safety, extra time needed for parenting activities, transportation, handling other people's reactions, providing proper discipline, and missing out visually. Blind people sense a reluctance on the part of others to acknowledge that they have a right to be parents. However, blind parents are fully capable of caring for their infants.

Which factors influence cervical dilation? (Select all that apply.) a. Strong uterine contractions b. Force of the presenting fetal part against the cervix c. Size of the woman d. Pressure applied by the amniotic sac e. Scarring of the cervix

A, B, D, E Dilation of the cervix occurs by the drawing upward of the musculofibrous components of the cervix, which is caused by strong uterine contractions. Pressure exerted by the amniotic fluid while the membranes are intact or by the force applied by the presenting part can also promote cervical dilation. Scarring of the cervix as a result of a previous infection or surgery may slow cervical dilation. Pelvic size or the size of the woman does not affect cervical dilation.

The transition to parenting for same-sex couples can present unique challenges. How can the nurse foster adjustment to parenting for these clients? (Select all that apply.) a. Use a supplemental feeding device to simulate breastfeeding. b. Allow the partner to cut the cord. c. Gay fathers should meet their new infant soon after the birth mother has recovered. d. Understand that strong social sanctions remain. e. Provide information regarding support groups.

A, B, D, E In a lesbian couple, the nonchildbearing partner may have a desire to breastfeed. This can be achieved using a supplemental nursing device. The female partner should be offered the same right as a heterosexual partner including cutting the cord. A gay couple may adopt a baby or use a surrogate. If the latter method is chosen, then they should be present at the birth if at all possible. The nurse can refer these men to available support groups. Same-sex couples continue to face strong social sanction in their efforts to parent.

Which practices contribute to the prevention of postpartum infection? (Select all that apply.) a. Not allowing the mother to walk barefoot at the hospital b. Educating the client to wipe from back to front after voiding c. Having staff members with conditions such as strep throat, conjunctivitis, and diarrhea stay home d. Instructing the mother to change her perineal pad from front to back each time she voids or defecates e. Not permitting visitors with cough or colds to enter the postpartum unit

A, C, D Proper perineal care helps prevent infection and aids in the healing process. Educating the woman to wipe from front to back (urethra to anus) after voiding or defecating is a simple first step. Walking barefoot and getting back into bed can contaminate the linens. Clients should wear shoes or slippers. Staff members with infections need to stay home until they are no longer contagious. The client should also wash her hands before and after these functions. Visitors with any signs of illness should not be allowed entry to the postpartum unit.

Which changes take place in the woman's reproductive system, days or even weeks before the commencement of labor? (Select all that apply.) a. Lightening b. Exhaustion c. Bloody show d. Rupture of membranes e. Decreased fetal movement

A, C, D Signs that precede labor may include lightening, urinary frequency, backache, weight loss, surge of energy, bloody show, and rupture of membranes. Many women experience a burst of energy before labor. A decrease in fetal movement is an ominous sign that does not always correlate with labor.

The class of drugs known as opioid analgesics (butorphanol, nalbuphine) is not suitable for administration to women with known opioid dependence. The antagonistic activity could precipitate withdrawal symptoms (abstinence syndrome) in both mothers and newborns. Which signs would indicate opioid or narcotic withdrawal in the mother? (Select all that apply.) a. Yawning, runny nose b. Increase in appetite c. Chills or hot flashes d. Constipation e. Irritability, restlessness

A, C, E The woman experiencing maternal opioid withdrawal syndrome will exhibit yawning, runny nose, sneezing, anorexia, chills or hot flashes, vomiting, diarrhea, abdominal pain, irritability, restlessness, muscle spasms, weakness, and drowsiness. Assessing both the mother and the newborn and planning the care accordingly are important steps for the nurse to take.

Nurses play a critical role in educating parents regarding measures to prevent infant abduction. Which instructions contribute to infant safety and security? (Select all that apply.) a. The mother should check the photo identification (ID) of any person who comes to her room. b. The baby should be carried in the parent's arms from the room to the nursery. c. Because of infant security systems, the baby can be left unattended in the client's room. d. Parents should use caution when posting photographs of their infant on the Internet. e. The mom should request that a second staff member verify the identity of any questionable person.

A, D, E Nurses must discuss infant security precautions with the mother and her family because infant abduction continues to be a concern. The mother should be taught to check the identity of any person who comes to remove the baby from her room. Hospital personnel usually wear picture identification patches. On some units, staff members also wear matching scrubs or special badges that are unique to the perinatal unit. As a rule, the baby is never carried in arms between the mother's room and the nursery, but rather the infant is always wheeled in a bassinet. The infant should never be left unattended, even if the facility has an infant security system. Parents should be instructed to use caution when posting photographs of their new baby on the Internet and on other public forums.

A parent who has a hearing impairment is presented with a number of challenges in parenting. Which nursing approaches are appropriate for working with hearing-impaired new parents? (Select all that apply.) a. Using devices that transform sound into light b. Assuming that the client knows sign language c. Speaking quickly and loudly d. Ascertaining whether the client can read lips before teaching e. Writing messages that aid in communication

A, D, E Section 504 of the Rehabilitation Act of 1973 requires that hospitals use various communication techniques and resources with the deaf and hard of hearing client. These resources include devices such as door alarms, cry alarms, and amplifiers. Before initiating communication, the nurse needs to be aware of the parents' preferences for communication. Not all hearing-impaired clients know sign language. Do they wear a hearing aid? Do they read lips? Do they wish to have a sign language interpreter? If the parent relies on lip reading, then the nurse should sit close enough to enable the parent to visualize lip movements. The nurse should speak clearly in a regular voice volume, in short, simple sentences. Written messages such as on a black or white erasable board can be useful. Written materials should be reviewed with the parents before discharge.

When caring for a newly delivered woman, what is the best measure to prevent abdominal distention after a cesarean birth? a. Rectal suppositories b. Early and frequent ambulation c. Tightening and relaxing abdominal muscles d. Carbonated beverages

B Activity will aid the movement of accumulated gas in the gastrointestinal tract. Rectal suppositories can be helpful after distention occurs; however, they do not prevent it. Ambulation is the best prevention. Carbonated beverages may increase distention.

A woman who is 39 weeks pregnant expresses fear about her impending labor and how she will manage. What is the nurse's ideal response? a. "Don't worry about it. You'll do fine." b. "It's normal to be anxious about labor. Let's discuss what makes you afraid." c. "Labor is scary to think about, but the actual experience isn't." d. "You can have an epidural. You won't feel anything."

B "It's normal to be anxious about labor. Let's discuss what makes you afraid" is a statement that allows the woman to share her concerns with the nurse and is a therapeutic communication tool. "Don't worry about it. You'll do fine" negates the woman's fears and is not therapeutic. "Labor is scary to think about, but the actual experience isn't" negates the woman's fears and offers a false sense of security. To suggest that every woman can have an epidural is untrue. A number of criteria must be met before an epidural is considered. Furthermore, many women still experience the feeling of pressure with an epidural.

Which statement by a newly delivered woman indicates that she knows what to expect regarding her menstrual activity after childbirth? a. "My first menstrual cycle will be lighter than normal and then will get heavier every month thereafter." b. "My first menstrual cycle will be heavier than normal and will return to my prepregnant volume within three or four cycles." c. "I will not have a menstrual cycle for 6 months after childbirth." d. "My first menstrual cycle will be heavier than normal and then will be light for several months after."

B "My first menstrual cycle will be heavier than normal and will return to my prepregnant volume within three or four cycles" is an accurate statement and indicates her understanding of her expected menstrual activity. She can expect her first menstrual cycle to be heavier than normal, which occurs by 3 months after childbirth, and the volume of her subsequent cycles will return to prepregnant levels within three to four cycles.

What is the correct terminology for the nerve block that provides anesthesia to the lower vagina and perineum? a. Epidural b. Pudendal c. Local d. Spinal block

B A pudendal block anesthetizes the lower vagina and perineum to provide anesthesia for an episiotomy and the use of low forceps, if needed. An epidural provides anesthesia for the uterus, perineum, and legs. A local provides anesthesia for the perineum at the site of the episiotomy. A spinal block provides anesthesia for the uterus, perineum, and down the legs.

Which statement concerning the third stage of labor is correct? a. The placenta eventually detaches itself from a flaccid uterus. b. An expectant or active approach to managing this stage of labor reduces the risk of complications. c. It is important that the dark, roughened maternal surface of the placenta appears before the shiny fetal surface. d. The major risk for women during the third stage is a rapid heart rate.

B Active management facilitates placental separation and expulsion, reducing the risk of complications. The placenta cannot detach itself from a flaccid (relaxed) uterus. Which surface of the placenta comes out first is not clinically important. The major risk for women during the third stage of labor is postpartum hemorrhaging.

Which client is most likely to experience strong and uncomfortable afterpains? a. A woman who experienced oligohydramnios b. A woman who is a gravida 4, para 4-0-0-4 c. A woman who is bottle-feeding her infant d. A woman whose infant weighed 5 pounds, 3 ounces

B Afterpains are more common in multiparous women. In a woman who experienced polyhydramnios, afterpains are more noticeable because the uterus was greatly distended. Breastfeeding may cause the afterpains to intensify. In a woman who delivered a large infant, afterpains are more noticeable because the uterus was greatly distended.

A woman who is gravida 3 para 2 arrives on the intrapartum unit. What is the most important nursing assessment at this time? a. Contraction pattern, amount of discomfort, and pregnancy history b. FHR, maternal vital signs, and the woman's nearness to birth c. Identification of ruptured membranes, woman's gravida and para, and her support person d. Last food intake, when labor began, and cultural practices the couple desires

B All options describe relevant intrapartum nursing assessments; however, this focused assessment has a priority. If the maternal and fetal conditions are normal and birth is not imminent, then other assessments can be performed in an unhurried manner; these include: gravida, para, support person, pregnancy history, pain assessment, last food intake, and cultural practices.

Under which circumstance would it be unnecessary for the nurse to perform a vaginal examination? a. Admission to the hospital at the start of labor b. When accelerations of the FHR are noted c. On maternal perception of perineal pressure or the urge to bear down d. When membranes rupture

B An accelerated FHR is a positive sign; therefore, a vaginal examination would not be necessary. A vaginal examination should be performed when the woman is admitted to the hospital, when she perceives perineal pressure or the urge to bear down, when her membranes rupture, when a significant change in her uterine activity has occurred, or when variable decelerations of the FHR are noted.

Anxiety is commonly associated with pain during labor. Which statement regarding anxiety is correct? a. Even mild anxiety must be treated. b. Severe anxiety increases tension, increases pain, and then, in turn, increases fear and anxiety, and so on. c. Anxiety may increase the perception of pain, but it does not affect the mechanism of labor. d. Women who have had a painful labor will have learned from the experience and have less anxiety the second time because of increased familiarity.

B Anxiety and pain reinforce each other in a negative cycle that will slow the progress of labor. Mild anxiety is normal for a woman in labor and likely needs no special treatment other than the standard reassurances. Anxiety increases muscle tension and ultimately can sufficiently build to slow the progress of labor. Unfortunately, an anxious, painful first labor is likely to carry over, through expectations and memories, into an anxious and painful experience in the second pregnancy.

A woman gave birth 48 hours ago to a healthy infant girl. She has decided to bottle feed. During the assessment, the nurse notices that both breasts are swollen, warm, and tender on palpation. Which guidance should the nurse provide to the client at this time? a. Run warm water on her breasts during a shower. b. Apply ice to the breasts for comfort. c. Express small amounts of milk from the breasts to relieve the pressure. d. Wearing a loose-fitting bra to prevent nipple irritation.

B Applying ice packs and cabbage leaves to the breasts for comfort is an appropriate intervention for treating engorgement in a mother who is bottle feeding. The ice packs should be applied for 15 minutes on and 45 minutes off to avoid rebound engorgement. A bottle-feeding mother should avoid any breast stimulation, including pumping or expressing milk. A bottle-feeding mother should continuously wear a well-fitted support bra or breast binder for at least the first 72 hours after giving birth. A loose-fitting bra will not aid lactation suppression. Furthermore, the shifting of the bra against the breasts may stimulate the nipples and thereby stimulate lactation.

A number of methods can be used for inducing labor. Which cervical ripening method falls under the category of mechanical or physical? a. Prostaglandins are used to soften and thin the cervix. b. Labor can sometimes be induced with balloon catheters or laminaria tents. c. Oxytocin is less expensive and more effective than prostaglandins but creates greater health risks. d. Amniotomy can be used to make the cervix more favorable for labor.

B Balloon catheters or laminaria tents are mechanical means of ripening the cervix. Ripening the cervix, making it softer and thinner, increases the success rate of induced labor. Prostaglandin E1 is less expensive and more effective than oxytocin but carries a greater risk. Amniotomy is the artificial rupture of membranes, which is used to induce labor only when the cervix is already ripe.

What three measures should the nurse implement to provide intrauterine resuscitation? a. Call the provider, reposition the mother, and perform a vaginal examination. b. Turn the client onto her side, provide oxygen (O2) via face mask, and increase intravenous (IV) fluids. c. Administer O2 to the mother, increase IV fluids, and notify the health care provider. d. Perform a vaginal examination, reposition the mother, and provide O2 via face mask.

B Basic interventions for the management of any abnormal FHR pattern include administering O2 via a nonrebreather face mask at a rate of 8 to 10 L/min, assisting the woman onto a side-lying (lateral) position, and increasing blood volume by increasing the rate of the primary IV infusion. The purpose of these interventions is to improve uterine blood flow and intervillous space blood flow and to increase maternal oxygenation and cardiac output. The term intrauterine resuscitation is sometimes used to refer to these interventions. If these interventions do not quickly resolve the abnormal FHR issue, then the primary provider should be immediately notified.

Developing a realistic birth plan with the pregnant woman regarding her care is important for the nurse. How would the nurse explain the major advantage of nonpharmacologic pain management? a. Greater and more complete pain relief is possible. b. No side effects or risks to the fetus are involved. c. The woman will remain fully alert at all times. d. Labor will likely be more rapid.

B Because nonpharmacologic pain management does not include analgesics, adjunct drugs, or anesthesia, it is harmless to the mother and the fetus. However, pain relief is lessened with nonpharmacologic pain management during childbirth. Although the woman's alertness is not altered by medication, the increase in pain may decrease alertness. Pain management may or may not alter the length of labor. At times when pain is decreased, the mother relaxes and labor progresses at a quicker pace.

The nurse who elects to work in the specialty of obstetric care must have the ability to distinguish between preterm birth, preterm labor, and low birth weight. Which statement regarding this terminology is correct? a. Terms preterm birth and low birth weight can be used interchangeably. b. Preterm labor is defined as cervical changes and uterine contractions occurring between 20 and 37 weeks of gestation. c. Low birth weight is a newborn who weighs below 3.7 pounds. d. Preterm birth rate in the United States continues to increase.

B Before 20 weeks of gestation, the fetus is not viable (miscarriage); after 37 weeks, the fetus can be considered term. Although these terms are used interchangeably, they have different meanings: preterm birth describes the length of gestation (before 37 weeks), regardless of the newborn's weight; low birth weight describes only the infant's weight at the time of birth (2500 g or less), whenever it occurs. Low birth weight is anything below 2500 g or approximately pounds. In 2011, the preterm birth rate in the United States was 11.7 %; it has dropped every year since 2008.

A primiparous woman is in the taking-in stage of psychosocial recovery and adjustment after childbirth. Recognizing the needs of women during this stage, how should the nurse respond? a. Foster an active role in the baby's care. b. Provide time for the mother to reflect on the events of her labor and delivery. c. Recognize the woman's limited attention span by giving her written materials to read when she gets home rather than doing a teaching session while she is in the hospital. d. Promote maternal independence by encouraging her to meet her own hygiene and comfort needs.

B During this stage, the new mother is excited and talkative. It is important that she be able to fulfill her desire to review her birth experience. During this stage, the new mother still relies upon others to meet her physical needs. Once these are met, she will be more able to take an active role, not only in her own care but also in the care of her newborn, which happens during the taking-hold stage. Short teaching sessions, using written materials to reinforce the content presented, is a more effective approach. The focus of the taking-in or dependency stage is to nurture the new mother by meeting her dependency needs for rest, comfort, hygiene, and nutrition.

A new client and her partner arrive on the labor, delivery, recovery, and postpartum (LDRP) unit for the birth of their first child. The nurse applies the electronic fetal monitor (EFM) to the woman. Her partner asks you to explain what is printing on the graph, referring to the EFM strip. He wants to know what the baby's heart rate should be. What is the nurse's best response? a. "Don't worry about that machine; that's my job." b. "The baby's heart rate will fluctuate in response to what is happening during labor." c. "The top line graphs the baby's heart rate, and the bottom line lets me know how strong the contractions are." d. "Your physician will explain all of that later."

B Explaining what indicates a normal FHR teaches the partner about fetal monitoring and provides support and information to alleviate his fears. Telling the partner not to worry discredits his feelings and does not provide the teaching he is requesting. Telling the partner that the graph indicates how strong the contractions are provides inaccurate information and does not address the partner's concerns about the FHR. The EFM graphs the frequency and duration of the contractions, not their intensity. Nurses should take every opportunity to provide teaching to the client and her family, especially when information is requested.

In follow-up appointments or visits with parents and their new baby, it may be useful if the nurse can identify parental behaviors that can either facilitate or inhibit attachment. Which is a facilitating behavior? a. Parents have difficulty naming the infant. b. Parents hover around the infant, directing attention to and pointing at the infant. c. Parents make no effort to interpret the actions or needs of the infant. d. Parents do not move from fingertip touch to palmar contact and holding.

B Hovering over the infant and obviously paying attention to the baby are facilitating behaviors. Inhibiting behaviors include difficulty naming the infant, making no effort to interpret the actions or needs of the infant, and not moving from fingertip touch to palmar contact and holding.

What is the primary difference between the labor of a nullipara and that of a multipara? a. Amount of cervical dilation b. Total duration of labor c. Level of pain experienced d. Sequence of labor mechanisms

B In a first-time pregnancy, the descent is usually slow but steady; in subsequent pregnancies, the descent is more rapid, resulting in a shorter duration of labor. Cervical dilation is the same for all labors. The level of pain is individual to the woman, not to the number of labors she has experienced. The sequence of labor mechanisms is the same with all labors.

In addition to eye contact, other early sensual contacts between the infant and mother involve sound and smell. What other statement regarding the senses is correct? a. High-pitched voices irritate newborns. b. Infants can learn to distinguish their mother's voice from others soon after birth. c. All babies in the hospital smell alike. d. Mother's breast milk has no distinctive odor.

B Infants know the sound of their mother's voice at an early age. Infants positively respond to high-pitched voices. Each infant has a unique odor. Infants quickly learn to distinguish the odor of their mother's breast milk.

Which characteristic correctly matches the type of deceleration with its likely cause? a. Early deceleration—umbilical cord compression b. Late deceleration—uteroplacental insufficiency c. Variable deceleration—head compression d. Prolonged deceleration—unknown cause

B Late deceleration is caused by uteroplacental insufficiency. Early deceleration is caused by head compression. Variable deceleration is caused by umbilical cord compression. Prolonged deceleration has a variety of either benign or critical causes.

What is the correct term describing the slight overlapping of cranial bones or shaping of the fetal head during labor? a. Lightening b. Molding c. Ferguson reflex d. Valsalva maneuver

B Molding also permits adaptation to various diameters of the maternal pelvis. Lightening is the mother's sensation of decreased abdominal distention, which usually occurs the week before labor. The Ferguson reflex is the contraction urge of the uterus after the stimulation of the cervix. The Valsalva maneuver describes conscious pushing during the second stage of labor.

What is the most likely cause for variable FHR decelerations? a. Altered fetal cerebral blood flow b. Umbilical cord compression c. Uteroplacental insufficiency d. Fetal hypoxemia

B Variable FHR decelerations can occur at any time during the uterine contracting phase and are caused by compression of the umbilical cord. Altered fetal cerebral blood flow results in early decelerations in the FHR. Uteroplacental insufficiency results in late decelerations in the FHR. Fetal hypoxemia initially results in tachycardia and then bradycardia if hypoxia continues.

What is the role of the nurse as it applies to informed consent? a. Inform the client about the procedure, and ask her to sign the consent form. b. Act as a client advocate, and help clarify the procedure and the options. c. Call the physician to see the client. d. Witness the signing of the consent form.

B Nurses play a part in the informed consent process by clarifying and describing procedures or by acting as the woman's advocate and asking the primary health care provider for further explanations. The physician is responsible for informing the woman of her options, explaining the procedure, and advising the client about potential risk factors. The physician must be present to explain the procedure to the client. However, the nurse's responsibilities go further than simply asking the physician to see the client. The nurse may witness the signing of the consent form. However, depending on the state's guidelines, the woman's husband or another hospital health care employee may sign as a witness.

According to professional standards (the Association of Women's Health, Obstetric and Neonatal Nurses [AWHONN], 2007), which action cannot be performed by the nonanesthetist registered nurse who is caring for a woman with epidural anesthesia? a. Monitoring the status of the woman and fetus b. Initiating epidural anesthesia c. Replacing empty infusion bags with the same medication and concentrate d. Stopping the infusion, and initiating emergency measures

B Only qualified, licensed anesthesia care providers are permitted to insert a catheter, initiate epidural anesthesia, verify catheter placement, inject medication through the catheter, or alter the medication or medications including type, amount, or rate of infusion. The nonanesthetist nurse is permitted to monitor the status of the woman, the fetus, and the progress of labor. Replacement of the empty infusion bags or syringes with the same medication and concentration is permitted. If the need arises, the nurse may stop the infusion, initiate emergency measures, and remove the catheter if properly educated to do so. Complications can require immediate interventions. Nurses must be prepared to provide safe and effective care during an emergency situation.

What should the laboring client who receives an opioid antagonist be told to expect? a. Her pain will decrease. b. Her pain will return. c. She will feel less anxious. d. She will no longer feel the urge to push.

B Opioid antagonists such as naloxone (Narcan) promptly reverse the CNS-depressant effects of opioids. In addition, the antagonist counters the effect of the stress-induced levels of endorphins. An opioid antagonist is especially valuable if the labor is more rapid than expected and the birth is anticipated when the opioid is at its peak effect. The woman should be told that the pain that was relieved by the opioid analgesic will return with the administration of the opioid antagonist. Her pain level will increase rather than decrease. Opioid antagonists have no effect on anxiety levels. They are primarily administered to reverse the excessive CNS depression in the mother, newborn, or both. An opioid antagonist (e.g., naloxone) has no effect on the mother's urge or ability to push. The practice of giving lower doses of IV opioids has reduced the incidence and severity of opioid-induced CNS depression; therefore, opioid antagonists are used less frequently.

What is the rationale for the administration of an oxytocic (e.g., Pitocin, Methergine) after expulsion of the placenta? a. To relieve pain b. To stimulate uterine contraction c. To prevent infection d. To facilitate rest and relaxation

B Oxytocics stimulate uterine contractions, which reduce blood loss after the third stage of labor. Oxytocics are not used to treat pain, do not prevent infection, and do not facilitate rest and relaxation.

The nurse observes that a first-time mother appears to ignore her newborn. Which strategy should the nurse use to facilitate mother-infant attachment? a. Tell the mother she must pay attention to her infant. b. Show the mother how the infant initiates interaction and attends to her. c. Demonstrate for the mother different positions for holding her infant while feeding. d. Arrange for the mother to watch a video on parent-infant interaction.

B Pointing out the responsiveness of the infant is a positive strategy for facilitating parent-infant attachment. Telling the mother that she must pay attention to her infant may be perceived as derogatory and is not appropriate. Educating the young mother in infant care is important, but pointing out the responsiveness of her baby is a better tool for facilitating mother-infant attachment. Videos are an educational tool that can demonstrate parent-infant attachment, but encouraging the mother to recognize the infant's responsiveness is more appropriate.

The postpartum nurse should be cognizant of what with regard to the adaptation of other family members (primarily siblings and grandparents) to the newborn? a. Sibling rivalry cannot be dismissed as overblown psychobabble; negative feelings and behaviors can take a long time to blow over. b. Participation in preparation classes helps both siblings and grandparents. c. In the United States, paternal and maternal grandparents consider themselves of equal importance and status. d. Since 1990, the number of grandparents providing permanent care to their grandchildren has been declining.

B Preparing older siblings, as well as grandparents, helps with everyone's adaptation. Sibling rivalry should be initially expected, but the negative behaviors associated with it have been overemphasized and stop in a comparatively short time. In the United States, in contrast to other cultures, paternal grandparents frequently consider themselves secondary to maternal grandparents. The number of grandparents providing permanent child care has been rising.

The postpartum woman continually repeats the story of her labor, delivery, and recovery experience. What is this new mother attempting to achieve with this behavior? a. Providing others with her knowledge of events b. Making the birth experience real c. Taking hold of the events leading up to her labor and delivery d. Accepting her response to labor and delivery

B Reliving the birth experience makes the event real and helps the mother realize that the pregnancy is over and that the infant is born and is now a separate individual. The retelling of the story satisfies her needs, not the needs of others. This new mother is in the taking-in phase, trying to make the birth experience seem real and separate the infant from herself.

The nurse observes several interactions between a postpartum woman and her new son. What behavior, if exhibited by this woman, would the nurse identify as a possible maladaptive behavior regarding parent-infant attachment? a. The postpartum woman talks and coos to her son. b. She seldom makes eye contact with her son. c. The mother cuddles her son close to her. d. She tells visitors how well her son is feeding.

B The mother should be encouraged to hold her infant in the en face position and make eye contact with the infant. Normal infant-parent interactions include talking and cooing to her son, cuddling her son close to her, and telling visitors how well her son is feeding.

The nurse should be cognizant of which postpartum physiologic alteration? a. Cardiac output, pulse rate, and stroke volume all return to prepregnancy normal values within a few hours of childbirth. b. Respiratory function returns to nonpregnant levels by 6 to 8 weeks after childbirth. c. Lowered white blood cell count after pregnancy can lead to false-positive results on tests for infections. d. Hypercoagulable state protects the new mother from thromboembolism, especially after a cesarean birth.

B Respirations should decrease to within the woman's normal prepregnancy range by 6 to 8 weeks after childbirth. Stroke volume increases and cardiac output remains high for a couple of days. However, the heart rate and blood pressure quickly return to normal. Leukocytosis increases 10 to 12 days after childbirth, which can obscure the diagnosis of acute infections, producing false-negative test results. The hypercoagulable state increases the risk of thromboembolism, especially after a cesarean birth.

A first-time mother is concerned about the type of medications she will receive during labor. The client is in a fair amount of pain and is nauseated. In addition, she appears to be very anxious. The nurse explains that opioid analgesics are often used along with sedatives. How should the nurse phrase the rationale for this medication combination? a. "The two medications, together, reduce complications." b. "Sedatives enhance the effect of the pain medication." c. "The two medications work better together, enabling you to sleep until you have the baby." d. "This is what your physician has ordered for you."

B Sedatives may be used to reduce the nausea and vomiting that often accompany opioid use. In addition, some ataractic drugs reduce anxiety and apprehension and potentiate the opioid analgesic affects. A potentiator may cause two drugs to work together more effectively, but it does not ensure zero maternal or fetal complications. Sedation may be a related effect of some ataractic drugs; however, sedation is not the goal. Furthermore, a woman is unlikely to be able to sleep through transitional labor and birth. Although the physician may have ordered the medication, "This is what your physician has ordered for you" is not an acceptable comment for the nurse to make.

After an emergency birth, the nurse encourages the woman to breastfeed her newborn. What is the primary purpose of this activity? a. To facilitate maternal-newborn interaction b. To stimulate the uterus to contract c. To prevent neonatal hypoglycemia d. To initiate the lactation cycle

B Stimulation of the nipples through breastfeeding or manual stimulation causes the release of oxytocin and prevents maternal hemorrhage. Breastfeeding facilitates maternal-newborn interaction, but it is not the primary reason a woman is encouraged to breastfeed after an emergency birth. The primary intervention for preventing neonatal hypoglycemia is thermoregulation. Cold stress can result in hypoglycemia. The woman is encouraged to breastfeed after an emergency birth to stimulate the release of oxytocin, which prevents hemorrhaging. Breastfeeding is encouraged to initiate the lactation cycle, but it is not the primary reason for this activity after an emergency birth.

Many first-time parents do not plan on having their parents' help immediately after the newborn arrives. Which statement by the nurse is the most appropriate when counseling new parents regarding the involvement of grandparents? a. "You should tell your parents to leave you alone." b. "Grandparents can help you with parenting skills." c. "Grandparent involvement can be very disruptive to the family." d. "They are getting old. You should let them be involved while they can."

B Telling the parents that grandparents can help with parenting skills and therefore help preserve family traditions is the most appropriate response. Intergenerational help may be perceived as interference, but telling the parents that their parents should be told to leave them alone is not therapeutic to the adaptation of the family. Telling the parents that grandparent involvement can be disruptive to the family is an invalid statement and not an appropriate nursing response. Regardless of age, grandparents can help with parenting skills and preserve family traditions.

Which nursing intervention is paramount when providing care to a client with preterm labor who has received terbutaline? a. Assess deep tendon reflexes (DTRs). b. Assess for dyspnea and crackles. c. Assess for bradycardia. d. Assess for hypoglycemia.

B Terbutaline is a beta2-adrenergic agonist that affects the mother's cardiopulmonary and metabolic systems. Signs of cardiopulmonary decompensation include adventitious breath sounds and dyspnea. An assessment for dyspnea and crackles is important for the nurse to perform if the woman is taking magnesium sulfate. Assessing DTRs does not address the possible respiratory side effects of using terbutaline. Since terbutaline is a beta2-adrenergic agonist, it can lead to hyperglycemia, not hypoglycemia. Beta2-adrenergic agonist drugs cause tachycardia, not bradycardia.

Nurses can help their clients by keeping them informed about the distinctive stages of labor. Which description of the phases of the first stage of labor is accurate? a. Latent: Mild, regular contractions; no dilation; bloody show; duration of 2 to 4 hours b. Active: Moderate, regular contractions; 4- to 7-cm dilation; duration of 3 to 6 hours c. Lull: No contractions; dilation stable; duration of 20 to 60 minutes d. Transition: Very strong but irregular contractions; 8- to 10-cm dilation; duration of 1 to 2 hours

B The active phase is characterized by moderate, regular contractions; 4- to 7-cm dilation; and a duration of 3 to 6 hours. The latent phase is characterized by mild-to-moderate and irregular contractions; dilation up to 3 cm; brownish-to-pale pink mucus, and a duration of 6 to 8 hours. No official "lull" phase exists in the first stage. The transition phase is characterized by strong- to-very strong and regular contractions; 8- to 10-cm dilation; and a duration of 20 to 40 minutes.

The nurse should be aware of what important information regarding systemic analgesics administered during labor? a. Systemic analgesics cross the maternal blood-brain barrier as easily as they do the fetal blood-brain barrier. b. Effects on the fetus and newborn can include decreased alertness and delayed sucking. c. Intramuscular (IM) administration is preferred over IV administration. d. IV patient-controlled analgesia (PCA) results in increased use of an analgesic.

B The effects of analgesics depend on the specific drug administered, the dosage, and the timing. Systemic analgesics cross the fetal blood-brain barrier more readily than the maternal blood-brain barrier. IV administration is preferred over IM administration because the drug acts faster and more predictably. PCA results in a decrease in the use of an analgesic.

The nurse observes a sudden increase in variability on the ERM tracing. Which class of medications may cause this finding? a. Narcotics b. Barbiturates c. Methamphetamines d. Tranquilizers

C Narcotics, barbiturates, and tranquilizers may be causes of decreased variability; whereas methamphetamines may cause increased variability.

A woman who has a history of sexual abuse may have a number of traumatic memories triggered during labor. She may fight the labor process and react with pain or anger. The nurse can implement a number of care measures to help her client view the childbirth experience in a positive manner. Which intervention is key for the nurse to use while providing care? a. Tell the client to relax and that it won't hurt much. b. Limit the number of procedures that invade her body. c. Reassure the client that, as the nurse, you know what is best. d. Allow unlimited care providers to be with the client.

B The number of invasive procedures such as vaginal examinations, internal monitoring, and IV therapy should be limited as much as possible. The nurse should always avoid words and phrases that may result in the client's recalling the phrases of her abuser (i.e., "Relax, this won't hurt" or "Just open your legs"). The woman's sense of control should be maintained at all times. The nurse should explain procedures at the client's pace and wait for permission to proceed. Protecting the client's environment by providing privacy and limiting the number of staff who observe the client will help to make her feel safe.

Which statement related to fetal positioning during labor is correct and important for the nurse to understand? a. Position is a measure of the degree of descent of the presenting part of the fetus through the birth canal. b. Birth is imminent when the presenting part is at +4 to +5 cm below the spine. c. The largest transverse diameter of the presenting part is the suboccipitobregmatic diameter. d. Engagement is the term used to describe the beginning of labor.

B The station of the presenting part should be noted at the beginning of labor to determine the rate of descent. Position is the relationship of the presenting part of the fetus to the four quadrants of the mother's pelvis; station is the measure of degree of descent. The largest diameter is usually the biparietal diameter. The suboccipitobregmatic diameter is the smallest, although one of the most critical. Engagement often occurs in the weeks just before labor in nulliparous women and before or during labor in multiparous women.

The nurse has received a report regarding a client in labor. The woman's last vaginal examination was recorded as 3 cm, 30%, and -2. What is the nurse's interpretation of this assessment? a. Cervix is effaced 3 cm and dilated 30%; the presenting part is 2 cm above the ischial spines. b. Cervix is dilated 3 cm and effaced 30%; the presenting part is 2 cm above the ischial spines. c. Cervix is effaced 3 cm and dilated 30%; the presenting part is 2 cm below the ischial spines. d. Cervix is dilated 3 cm and effaced 30%; the presenting part is 2 cm below the ischial spines.

B The sterile vaginal examination is recorded as centimeters of cervical dilation, percentage of cervical dilation, and the relationship of the presenting part to the ischial spines (either above or below). For this woman, the cervix is dilated 3 cm and effaced 30%, and the presenting part is 2 cm above the ischial spines. The first interpretation of this vaginal examination is incorrect; the cervix is dilated 3 cm and is 30% effaced. However, the presenting part is correct at 2 cm above the ischial spines. The remaining two interpretations of this vaginal examination are incorrect. Although the dilation and effacement are correct at 3 cm and 30%, the presenting part is actually 2 cm above the ischial spines.

What is the rationale for the use of a blood patch after spinal anesthesia? a. Hypotension b. Headache c. Neonatal respiratory depression d. Loss of movement

B The subarachnoid block may cause a postspinal headache resulting from the loss of cerebrospinal fluid from the puncture in the dura. When blood is injected into the epidural space in the area of the dural puncture, it forms a seal over the hole to stop the leaking of cerebrospinal fluid. Hypotension is prevented by increasing fluid volume before the procedure. Neonatal respiratory depression is not an expected outcome with spinal anesthesia. Loss of movement is an expected outcome of spinal anesthesia.

Which statement regarding the postpartum uterus is correct? a. At the end of the third stage of labor, the postpartum uterus weighs approximately 500 g. b. After 2 weeks postpartum, it should be abdominally nonpalpable. c. After 2 weeks postpartum, it weighs 100 g. d. Postpartum uterus returns to its original (prepregnancy) size by 6 weeks postpartum.

B The uterus does not return to its original size. At the end of the third stage of labor, the uterus weighs approximately 1000 g. After 2 weeks postpartum, the uterus weighs approximately 350 g. The normal self-destruction of excess hypertrophied tissue accounts for the slight increase in uterine size after each pregnancy.

A pregnant woman at 29 weeks of gestation has been diagnosed with preterm labor. Her labor is being controlled with tocolytic medications. She asks when she might be able to go home. Which response by the nurse is most accurate? a. "After the baby is born." b. "When we can stabilize your preterm labor and arrange home health visits." c. "Whenever your physician says that it is okay." d. "It depends on what kind of insurance coverage you have.

B This client's preterm labor is being controlled with tocolytics. Once she is stable, home care may be a viable option for this type of client. Care of a client with preterm labor is multidisciplinary and multifactorial; the goal is to prevent delivery. In many cases, this goal may be achieved at home. Managed care may dictate an earlier hospital discharge or a shift from hospital to home care. Insurance coverage may be one factor in client care, but ultimately, client safety remains the most important factor.

Through a vaginal examination, the nurse determines that a woman is 4 cm dilated. The external fetal monitor shows uterine contractions every to 4 minutes. The nurse reports this as what stage of labor? a. First stage, latent phase b. First stage, active phase c. First stage, transition phase d. Second stage, latent phase

B This maternal progress indicates that the woman is in the active phase of the first stage of labor. During the latent phase of the first stage of labor, the expected maternal progress is 0 to 3 cm dilation with contractions every 5 to 30 minutes. During the transition phase of the first stage of labor, the expected maternal progress is 8 to 10 cm dilation with contractions every 2 to 3 minutes. During the latent phase of the second stage of labor, the woman is completely dilated and experiences a restful period of "laboring down."

Which description of postpartum restoration or healing times is accurate? a. The cervix shortens, becomes firm, and returns to form within a month postpartum. b. Vaginal rugae reappear by 3 weeks postpartum. c. Most episiotomies heal within a week. d. Hemorrhoids usually decrease in size within 2 weeks of childbirth.

B Vaginal rugae reappear by 3 weeks postpartum; however, they are never as prominent as in nulliparous women. The cervix regains its form within days; the cervical os may take longer. Most episiotomies take 2 to 3 weeks to heal. Hemorrhoids can take 6 weeks to decrease in size.

What information should the nurse understand fully regarding rubella and Rh status? a. Breastfeeding mothers cannot be vaccinated with the live attenuated rubella virus. b. Women should be warned that the rubella vaccination is teratogenic and that they must avoid pregnancy for at least 1 month after vaccination. c. Rh immunoglobulin is safely administered intravenously because it cannot harm a nursing infant. d. Rh immunoglobulin boosts the immune system and thereby enhances the effectiveness of vaccinations.

B Women should understand that they must practice contraception for at least 1 month after being vaccinated. Because the live attenuated rubella virus is not communicable in breast milk, breastfeeding mothers can be vaccinated. Rh immunoglobulin is administered intramuscular (IM); it should never be administered to an infant. Rh immunoglobulin suppresses the immune system and therefore might thwart the rubella vaccination.

In the United States, the en face position is preferred immediately after birth. Which actions by the nurse can facilitate this process? (Select all that apply.) a. Washing both the infant's face and the mother's face b. Placing the infant on the mother's abdomen or breast with their heads on the same plane c. Dimming the lights d. Delaying the instillation of prophylactic antibiotic ointment in the infant's eyes e. Placing the infant in the grandmother's arms

B, C, D As newborns become functionally able to sustain eye contact with their parents, they spend time in mutual gazing, often in the en face position, a position in which the faces of the parent and infant are approximately 20 cm apart and on the same plane. Washing the faces of the infant or mother is not necessary at this time and would interrupt the process. Nurses and physicians or midwives can facilitate eye contact immediately after birth by placing the infant on the mother's abdomen or breasts with the mother and the infant's faces on the same plane. Dimming the lights encourages the infant's eyes to stay open. To promote eye contact, the instillation of prophylactic antibiotic ointment into the infant's eyes can be delayed until after the infant and parents have had some time together during the first hour after birth. Having the grandmother hold the infant is important; however, it will not necessarily promote eye contact between the parent and infant.

The breast-feeding mother should be taught to expect which changes to the condition of the breasts? (Select all that apply.) a. Breast tenderness is likely to persist for approximately 1 week after the start of lactation. b. As lactation is established, a mass may form that can be distinguished from cancer by its positional shift from day to day. c. In nonlactating mothers, colostrum is present for the first few days after childbirth. d. If suckling is never begun or is discontinued, then lactation ceases within a few days to a week. e. Little change occurs to the breasts in the first 48 hours.

B, C, D Breasts become fuller and heavier as colostrum transitions to milk; this fullness should last 72 to 96 hours. The movable, noncancerous mass is a filled milk sac. Colostrum is present for a few days whether or not the mother breastfeeds. A mother who does not want to breastfeed should also avoid stimulating her nipples. Little change to the breasts occurs in the first 24 hours of childbirth.

Maternal hypotension is a potential side effect of regional anesthesia and analgesia. What nursing interventions could the nurse use to increase the client's blood pressure? (Select all that apply.) a. Place the woman in a supine position. b. Place the woman in a lateral position. c. Increase IV fluids. d. Administer oxygen. e. Perform a vaginal examination.

B, C, D Nursing interventions for maternal hypotension arising from analgesia or anesthesia include turning the woman to a lateral position, increasing IV fluids, administering oxygen via face mask, elevating the woman's legs, notifying the physician, administering an IV vasopressor, and monitoring the maternal and fetal status at least every 5 minutes until the woman is stable. Placing the client in a supine position causes venous compression, thereby limiting blood flow to and oxygenation of the placenta and fetus. A sterile vaginal examination has no bearing on maternal blood pressure.

Which FHR decelerations would require the nurse to change the maternal position? (Select all that apply.) a. Early decelerations b. Late decelerations c. Variable decelerations d. Moderate decelerations e. Prolonged decelerations

B, C, E Early decelerations (and accelerations) do not generally need any nursing intervention. Late decelerations suggest that the nurse should change the maternal position (lateral). Variable decelerations also require a maternal position change (side to side). Moderate decelerations are not an accepted category. Prolonged decelerations are late or variable decelerations that last for a prolonged period (longer than 2 minutes) and require intervention.

Indications for a primary cesarean birth are often nonrecurring. Therefore, a woman who has had a cesarean birth with a low transverse scar may be a candidate for vaginal birth after cesarean (VBAC). Which clients would be less likely to have a successful VBAC? (Select all that apply.) a. Lengthy interpregnancy interval b. African-American race c. Delivery at a rural hospital d. Estimated fetal weight <4000 g e. Maternal obesity (BMI >30)

B, C, E Indications for a low success rate for a VBAC delivery include a short interpregnancy interval, non-Caucasian race, gestational age longer than 40 weeks, maternal obesity, preeclampsia, fetal weight greater than 4000 g, and delivery at a rural or private hospital.

A tiered system of categorizing FHR has been recommended by professional organizations. Nurses, midwives, and physicians who care for women in labor must have a working knowledge of fetal monitoring standards and understand the significance of each category. What is the correct nomenclature for these categories? (Select all that apply.) a. Reassuring b. Category I c. Category II d. Nonreassuring e. Category III

B, C, E The three-tiered system of FHR tracings include category I, II, and III. Category I is a normal tracing requiring no action. Category II FHR tracings are indeterminate and includes tracings that do not meet category I or III criteria. Category III tracings are abnormal and require immediate intervention.

Postpartum fatigue (PPF) is more than just feeling tired. It is a complex phenomenon affected by physiologic, psychologic, and situational variables. Which factors contribute to this phenomenon? (Select all that apply.) a. Precipitous labor b. Hospital routines c. Bottle feeding d. Anemia e. Excitement

B, D, E Physical fatigue and exhaustion are often associated with a long labor or cesarean birth, hospital routines, breastfeeding, and infant care. PPF is also attributed to anemia, infection, or thyroid dysfunction. The excitement and exhilaration of delivering a new infant along with well-intentioned visitors may make rest difficult.

The nurse recognizes that uterine hyperstimulation with oxytocin requires emergency interventions. What clinical cues alert the nurse that the woman is experiencing uterine hyperstimulation? (Select all that apply.) a. Uterine contractions lasting <90 seconds and occurring >2 minutes in frequency b. Uterine contractions lasting >90 seconds and occurring <2 minutes in frequency c. Uterine tone <20 mm Hg d. Uterine tone >20 mm Hg e. Increased uterine activity accompanied by a nonreassuring FHR and pattern

B, D, E Uterine contractions that occur less frequently than 2 minutes apart and last longer than 90 seconds, a uterine tone over 20 mm Hg, and a nonreassuring FHR and pattern are indications of uterine hyperstimulation with oxytocin administration. Uterine contractions that occur more frequently than 2 minutes apart and last less than 90 seconds are the expected goal of oxytocin induction. A uterine tone less than 20 mm Hg is normal.

According to the National Institute of Child Health and Human Development (NICHD) Three-Tier System of FHR Classification, category III tracings include all FHR tracings not categorized as category I or II. Which characteristics of the FHR belong in category III? (Select all that apply.) a. Baseline rate of 110 to 160 beats per minute b. Tachycardia c. Absent baseline variability not accompanied by recurrent decelerations d. Variable decelerations with other characteristics such as shoulders or overshoots e. Absent baseline variability with recurrent variable decelerations f. Bradycardia

B, D, E, F Tachycardia, variable decelerations with other characteristics, absent baseline variability with recurrent variable decelerations, and bradycardia are characteristics that are considered nonreassuring or abnormal and belong in category III. A FHR of 110 to 160 beats per minute is considered normal and belongs in category I. Absent baseline variability not accompanied by recurrent decelerations is a category II characteristic.

Which nursing assessment indicates that a woman who is in second-stage labor is almost ready to give birth? a. Fetal head is felt at 0 station during vaginal examination. b. Bloody mucous discharge increases. c. Vulva bulges and encircles the fetal head. d. Membranes rupture during a contraction.

C A bulging vulva that encircles the fetal head describes crowning, which occurs shortly before birth. Birth of the head occurs when the station is +4. A 0 station indicates engagement. Bloody show occurs throughout the labor process and is not an indication of an imminent birth. ROM can occur at any time during the labor process and does not indicate an imminent birth.

A woman in labor is breathing into a mouthpiece just before the start of her regular contractions. As she inhales, a valve opens and gas is released. She continues to inhale the gas slowly and deeply until the contraction starts to subside. When the inhalation stops, the valve closes. Which statement regarding this procedure is correct? a. The application of nitrous oxide gas is not often used anymore. b. An inhalation of gas is likely to be used in the second stage of labor, not during the first stage. c. An application of nitrous oxide gas is administered for pain relief. d. The application of gas is a prelude to a cesarean birth.

C A mixture of nitrous oxide with oxygen in a low concentration can be used in combination with other nonpharmacologic and pharmacologic measures for pain relief. This procedure is still commonly used in Canada and in the United Kingdom. Nitrous oxide inhaled in a low concentration will reduce but not eliminate pain during the first and second stages of labor. Nitrous oxide inhalation is not generally used before a caesarean birth. Nitrous oxide does not appear to depress uterine contractions or cause adverse reactions in the newborn.

Which statement is not an expected outcome for the client who attends a reputable childbirth preparation program? a. Childbirth preparation programs increase the woman's sense of control. b. Childbirth preparation programs prepare a support person to help during labor. c. Childbirth preparation programs guarantee a pain-free childbirth. d. Childbirth preparation programs teach distraction techniques.

C All methods try to increase a woman's sense of control, prepare a support person, and train the woman in physical conditioning, which includes breathing techniques. These programs cannot, and reputable ones do not, promise a pain-free childbirth. Increasing a woman's sense of control is the goal of all childbirth preparation methods. Preparing a support person to help in labor is a vitally important component of any childbirth education program. The coach may learn how to touch a woman's body to detect tense and contracted muscles. The woman then learns how to relax in response to the gentle stroking by the coach. Distraction techniques are a form of care that are effective to some degree in relieving labor pain and are taught in many childbirth programs. These distractions include imagery, feedback relaxation, and attention-focusing behaviors.

The nurse is performing an initial assessment of a client in labor. What is the appropriate terminology for the relationship of the fetal body parts to one another? a. Lie b. Presentation c. Attitude d. Position

C Attitude is the relationship of the fetal body parts to one another. Lie is the relationship of the long axis (spine) of the fetus to the long axis (spine) of the mother. Presentation refers to the part of the fetus that enters the pelvic inlet first and leads through the birth canal during labor at term. Position is the relationship of the presenting part of the fetus to the four quadrants of the mother's pelvis.

What is the primary purpose for the use of tocolytic therapy to suppress uterine activity? a. Drugs can be efficaciously administered up to the designated beginning of term at 37 weeks gestation. b. Tocolytic therapy has no important maternal (as opposed to fetal) contraindications. c. The most important function of tocolytic therapy is to provide the opportunity to administer antenatal glucocorticoids. d. If the client develops pulmonary edema while receiving tocolytic therapy, then intravenous (IV) fluids should be given.

C Buying time for antenatal glucocorticoids to accelerate fetal lung development may be the best reason to use tocolytic therapy. Once the pregnancy has reached 34 weeks, however, the risks of tocolytic therapy outweigh the benefits. Important maternal contraindications to tocolytic therapy exist. Tocolytic-induced edema can be caused by IV fluids.

A nurse hears a primiparous woman talking to her son and telling him that his chin is just like his dad's. This statement is most descriptive of which process? a. Mutuality b. Synchrony c. Claiming d. Reciprocity

C Claiming refers to the process by which the child is identified in terms of likeness to other family members. Mutuality occurs when the infant's behaviors and characteristics call forth a corresponding set of maternal behaviors and characteristics. Synchrony refers to the "fit" between the infant's cues and the parent's responses. Reciprocity is a type of body movement or behavior that provides the observer with cues.

Which nursing assessment indicates that a woman who is in second-stage labor is almost ready to give birth? a. Fetal head is felt at 0 station during the vaginal examination. b. Bloody mucous discharge increases. c. Vulva bulges and encircles the fetal head. d. Membranes rupture during a contraction.

C During the active pushing (descent) phase, the woman has strong urges to bear down as the presenting part of the fetus descends and presses on the stretch receptors of the pelvic floor. The vulva stretches and begins to bulge, encircling the fetal head. Birth of the head occurs when the station is +4. A 0 station indicates engagement. Bloody show occurs throughout the labor process and is not an indication of an imminent birth. Rupture of membranes can occur at any time during the labor process and does not indicate an imminent birth.

Which clinical finding indicates that the client has reached the second stage of labor? a. Amniotic membranes rupture. b. Cervix cannot be felt during a vaginal examination. c. Woman experiences a strong urge to bear down. d. Presenting part of the fetus is below the ischial spines.

C During the descent phase of the second stage of labor, the woman may experience an increase in the urge to bear down. The ROM has no significance in determining the stage of labor. The second stage of labor begins with full cervical dilation. Many women may have an urge to bear down when the presenting fetal part is below the level of the ischial spines. This urge can occur during the first stage of labor, as early as with 5 cm dilation.

What should the nurse's next action be if the client's white blood cell (WBC) count is 25,000/mm3 on her second postpartum day? a. Immediately inform the physician. b. Have the laboratory draw blood for reanalysis. c. Recognize that this count is an acceptable range at this point postpartum. d. Immediately begin antibiotic therapy.

C During the first 10 to 12 days after childbirth, WBC values between 20,000 and 25,000/mm3 are common. Because a WBC count of 25,000/mm3 on her second postpartum day is normal, alerting the physician is not warranted nor is reassessment or antibiotics needed; the WBC count is not elevated.

During a telephone follow-up conversation with a woman who is 4 days postpartum, the woman tells the nurse, "I don't know what's wrong. I love my son, but I feel so let down. I seem to cry for no reason!" Which condition might this new mother be experiencing? a. Letting-go b. Postpartum depression (PPD) c. Postpartum blues d. Attachment difficulty

C During the postpartum blues, women are emotionally labile, often crying easily and for no apparent reason. This lability seems to peak around the fifth postpartum day. The letting-go phase is the period that occurs several weeks after childbirth. During this phase the woman wants to move forward as a family unit with all members, appropriately interacting to their new roles. PPD is an intense, pervasive sadness marked by severe, labile mood swings; it is more serious and persistent than the postpartum blues. Crying is not a maladaptive attachment response; it indicates postpartum blues.

Which statement, related to the reconditioning of the urinary system after childbirth, should the nurse understand? a. Kidney function returns to normal a few days after birth. b. Diastasis recti abdominis is a common condition that alters the voiding reflex. c. Fluid loss through perspiration and increased urinary output accounts for a weight loss of more than 2 kg during the puerperium. d. With adequate emptying of the bladder, bladder tone is usually restored 2 to 3 weeks after childbirth.

C Excess fluid loss through other means besides perspiration and increased urinary output occurs as well. Kidney function usually returns to normal in approximately 1 month. Diastasis recti abdominis is the separation of muscles in the abdominal wall and has no effect on the voiding reflex. Bladder tone is usually restored 5 to 7 days after childbirth.

In which clinical situation would the nurse most likely anticipate a fetal bradycardia? a. Intraamniotic infection b. Fetal anemia c. Prolonged umbilical cord compression d. Tocolytic treatment using terbutaline

C Fetal bradycardia can be considered a later sign of fetal hypoxia and is known to occur before fetal death. Bradycardia can result from placental transfer of drugs, prolonged compression of the umbilical cord, maternal hypothermia, and maternal hypotension. Intraamniotic infection, fetal anemia, and tocolytic treatment using terbutaline would most likely result in fetal tachycardia.

When assessing the fetus using Leopold's maneuvers, the nurse feels a round, firm, and movable fetal part in the fundal portion of the uterus and a long, smooth surface in the mother's right side close to midline. What is the position of the fetus? a. ROA b. LSP c. RSA d. LOA

C Fetal position is denoted with a three-letter abbreviation. The first letter indicates the presenting part in either the right or the left side of the maternal pelvis. The second letter indicates the anatomic presenting part of the fetus. The third letter stands for the location of the presenting part in relationship to the anterior, posterior, or transverse portion of the maternal pelvis. Palpation of a round, firm fetal part in the fundal portion of the uterus would be the fetal head, indicating that the fetus is in a breech position with the sacrum as the presenting part in the maternal pelvis. Palpation of the fetal spine along the mother's right side denotes the location of the presenting part in the mother's pelvis. The ability to palpate the fetal spine indicates that the fetus is anteriorly positioned in the maternal pelvis. This fetus is anteriorly positioned in the right side of the maternal pelvis with the sacrum as the presenting part. RSA is the correct three-letter abbreviation to indicate this fetal position. ROA denotes a fetus that is anteriorly positioned in the right side of the maternal pelvis with the occiput as the presenting part. LSP describes a fetus that is posteriorly positioned in the left side of the pelvis with the sacrum as the presenting part. A fetus that is LOA would be anteriorly positioned in the left side of the pelvis with the occiput as the presenting part.

A woman's position is an important component of the labor progress. Which guidance is important for the nurse to provide to the laboring client? a. The supine position, which is commonly used in the United States, increases blood flow. b. The laboring client positioned on her hands and knees ("all fours" position) is hard on the woman's back. c. Frequent changes in position help relieve fatigue and increase the comfort of the laboring client. d. In a sitting or squatting position, abdominal muscles of the laboring client will have to work harder.

C Frequent position changes relieve fatigue, increase comfort, and improve circulation. Blood flow can be compromised in the supine position; any upright position benefits cardiac output. The "all fours" position is used to relieve backache in certain situations. In a sitting or squatting position, the abdominal muscles work in greater harmony with uterine contractions.

A recently delivered mother and her baby are at the clinic for a 6-week postpartum checkup. Which response by the client alerts the nurse that psychosocial outcomes have not been met? a. The woman excessively discusses her labor and birth experience. b. The woman feels that her baby is more attractive and clever than any others. c. The woman has not given the baby a name. d. The woman has a partner or family members who react very positively about the baby.

C If the mother is having difficulty naming her new infant, it may be a signal that she is not adapting well to parenthood. Other red flags include a refusal to hold or feed the baby, a lack of interaction with the infant, and becoming upset when the baby vomits or needs a diaper change. A new mother who is having difficulty is unwilling to discuss her labor and birth experience. An appropriate nursing diagnosis might be Impaired parenting, related to a long, difficult labor or unmet expectations of birth. A mother who is willing to discuss her birth experience is making a healthy personal adjustment. The mother who is not coping well finds her baby unattractive and messy. She may also be overly disappointed in the baby's sex. The client might voice concern that the baby reminds her of a family member whom she does not like. Having a partner and/or other family members react positively is an indication that this new mother has a good support system in place. This support system helps reduce anxiety related to her new role as a mother.

Postpartum overdistention of the bladder and urinary retention can lead to which complications? a. Postpartum hemorrhage and eclampsia b. Fever and increased blood pressure c. Postpartum hemorrhage and urinary tract infection d. Urinary tract infection and uterine rupture

C Incomplete emptying and overdistention of the bladder can lead to urinary tract infection. Overdistention of the bladder displaces the uterus and prevents contraction of the uterine muscle, thus leading to postpartum hemorrhage. No correlation exists between bladder distention and high blood pressure or eclampsia. The risk of uterine rupture decreases after the birth of the infant.

What is the primary rationale for the thorough drying of the infant immediately after birth? a. Stimulates crying and lung expansion b. Removes maternal blood from the skin surface c. Reduces heat loss from evaporation d. Increases blood supply to the hands and feet

C Infants are wet with amniotic fluid and blood at birth, and this accelerates evaporative heat loss. The primary purpose of drying the infant is to prevent heat loss. Although rubbing the infant stimulates crying, it is not the main reason for drying the infant. This process does not remove all the maternal blood.

The nurse who provides care to clients in labor must have a thorough understanding of the physiologic processes of maternal hypotension. Which outcome might occur if the interventions for maternal hypotension are inadequate? a. Early FHR decelerations b. Fetal arrhythmias c. Uteroplacental insufficiency d. Spontaneous rupture of membranes

C Low maternal blood pressure reduces placental blood flow during uterine contractions, resulting in fetal hypoxemia. Maternal hypotension does not result in early FHR decelerations nor is it associated with fetal arrhythmias. Spontaneous rupture of membranes is not a result of maternal hypotension.

Where is the point of maximal intensity (PMI) of the FHR located? a. Usually directly over the fetal abdomen b. In a vertex position, heard above the mother's umbilicus c. Heard lower and closer to the midline of the mother's abdomen as the fetus descends and internally rotates d. In a breech position, heard below the mother's umbilicus

C Nurses should be prepared for the shift. The PMI of the FHR is usually directly over the fetal back. In a vertex position, the PMI of the FHR is heard below the mother's umbilicus. In a breech position, it is heard above the mother's umbilicus.

The early postpartum period is a time of emotional and physical vulnerability. Many mothers can easily become psychologically overwhelmed by the reality of their new parental responsibilities. Fatigue compounds these issues. Although the baby blues are a common occurrence in the postpartum period, approximately 500,000 women in America experience a more severe syndrome known as PPD. Which statement regarding PPD is essential for the nurse to be aware of when attempting to formulate a nursing diagnosis? a. PPD symptoms are consistently severe. b. This syndrome affects only new mothers. c. PPD can easily go undetected. d. Only mental health professionals should teach new parents about this condition.

C PPD can go undetected because parents do not voluntarily admit to this type of emotional distress out of embarrassment, fear, or guilt. PPD symptoms range from mild to severe, with women having both good and bad days. PPD may also affect new fathers. Therefore, both mothers and fathers should be screened. The nurse should include information on PPD and how to differentiate it from the baby blues for all clients before discharge. Nurses can also urge new parents to report symptoms and to seek follow-up care promptly if symptoms occur.

Prostaglandin gel has been ordered for a pregnant woman at 43 weeks of gestation. What is the primary purpose of prostaglandin administration? a. To enhance uteroplacental perfusion in an aging placenta b. To increase amniotic fluid volume c. To ripen the cervix in preparation for labor induction d. To stimulate the amniotic membranes to rupture

C Preparations of prostaglandin E1 and E2 are effective when used before labor induction to ripen (i.e., soften and thin) the cervix. Uteroplacental perfusion is not altered by the use of prostaglandins. The insertion of prostaglandin gel has no effect on the level of amniotic fluid. In some cases, women will spontaneously begin laboring after the administration of prostaglandins, thereby eliminating the need for oxytocin. It is not common for a woman's membranes to rupture as a result of prostaglandin use.

Which hormone remains elevated in the immediate postpartum period of the breastfeeding woman? a. Estrogen b. Progesterone c. Prolactin d. Human placental lactogen

C Prolactin levels in the blood progressively increase throughout pregnancy. In women who breastfeed, prolactin levels remain elevated into the sixth week after birth. Estrogen levels decrease significantly after expulsion of the placenta, reaching their lowest levels 1 week into the postpartum period. Progesterone levels decrease significantly after expulsion of the placenta, reaching their lowest levels 1 week into the postpartum period. Human placental lactogen levels dramatically decrease after expulsion of the placenta.

In planning for home care of a woman with preterm labor, which concern should the nurse need to address? a. Nursing assessments are different from those performed in the hospital setting. b. Restricted activity and medications are necessary to prevent a recurrence of preterm labor. c. Prolonged bed rest may cause negative physiologic effects. d. Home health care providers are necessary.

C Prolonged bed rest may cause adverse effects such as weight loss, loss of appetite, muscle wasting, weakness, bone demineralization, decreased cardiac output, risk for thrombophlebitis, alteration in bowel functions, sleep disturbance, and prolonged postpartum recovery. Nursing assessments differ somewhat from those performed in the acute care setting, but this concern does not need to be addressed. Restricted activity and medications may prevent preterm labor but not in all women. In addition, the plan of care is individualized to meet the needs of each client. Many women receive home health nurse visits, but care is individualized for each woman.

Which information related to a prolonged deceleration is important for the labor nurse to understand? a. Prolonged decelerations present a continuing pattern of benign decelerations that do not require intervention. b. Prolonged decelerations constitute a baseline change when they last longer than 5 minutes. c. A disruption to the fetal oxygen supply causes prolonged decelerations. d. Prolonged decelerations require the customary fetal monitoring by the nurse.

C Prolonged decelerations are caused by a disruption in the fetal oxygen supply. They usually begin as a reflex response to hypoxia. If the disruption continues, then the fetal cardiac tissue, itself, will become hypoxic, resulting in direct myocardial depression of the FHR. Prolonged decelerations can be caused by prolonged cord compression, uteroplacental insufficiency, or perhaps sustained head compression. Prolonged decelerations lasting longer than 10 minutes are considered a baseline change that may require intervention. A prolonged deceleration is a visually apparent decrease (may be either gradual or abrupt) in the FHR of at least 15 beats per minute below the baseline and lasting longer than 2 minutes but shorter than 10 minutes. Nurses should immediately notify the physician or nurse-midwife and initiate appropriate treatment of abnormal patterns when they see prolonged decelerations.

Which statement by the client would lead the nurse to believe that labor has been established? a. "I passed some thick, pink mucus when I urinated this morning." b. "My bag of waters just broke." c. "The contractions in my uterus are getting stronger and closer together." d. "My baby dropped, and I have to urinate more frequently now."

C Regular, strong contractions with the presence of cervical change indicate that the woman is experiencing true labor. Although the loss of the mucous plug (operculum) often occurs during the first stage of labor or before the onset of labor, it is not the indicator of true labor. Spontaneous rupture of membranes often occurs during the first stage of labor; however, it is not an indicator of true labor. The presenting part of the fetus typically becomes engaged in the pelvis at the onset of labor but is not the indicator of true labor.

When would an internal version be indicated to manipulate the fetus into a vertex position? a. Fetus from a breech to a cephalic presentation before labor begins b. Fetus from a transverse lie to a longitudinal lie before a cesarean birth c. Second twin from an oblique lie to a transverse lie before labor begins d. Second twin from a transverse lie to a breech presentation during a vaginal birth

D Internal version is used only during a vaginal birth to manipulate the second twin into a presentation that allows it to be vaginally born. For internal version to occur, the cervix needs to be completely dilated.

Which statement by the client will assist the nurse in determining whether she is in true labor as opposed to false labor? a. "I passed some thick, pink mucus when I urinated this morning." b. "My bag of waters just broke." c. "The contractions in my uterus are getting stronger and closer together." d. "My baby dropped, and I have to urinate more frequently now."

C Regular, strong contractions with the presence of cervical change indicate that the woman is experiencing true labor. The loss of the mucous plug (operculum) often occurs during the first stage of labor or before the onset of labor, but it is not the indicator of true labor. Spontaneous rupture of membranes (ROM) often occurs during the first stage of labor, but it is not the indicator of true labor. The presenting part of the fetus typically becomes engaged in the pelvis at the onset of labor, but this is not the indicator of true labor.

The nurse is teaching a client with preterm premature rupture of membranes (PPROM) regarding self-care activities. Which activities should the nurse include in her teaching? a. Report a temperature higher than 40° C. b. Tampons are safe to use to absorb the leaking amniotic fluid. c. Do not engage in sexual activity. d. Taking frequent tub baths is safe.

C Sexual activity should be avoided because it may induce preterm labor. A temperature higher than 38° C should be reported. To prevent the risk of infection, tub baths should be avoided and nothing should be inserted into the vagina. Further, foul-smelling vaginal fluid, which may be a sign of infection, should be reported.

Which description of the phases of the second stage of labor is most accurate? a. Latent phase: Feeling sleepy; fetal station 2+ to 4+; duration of 30 to 45 minutes b. Active phase: Overwhelmingly strong contractions; Ferguson reflux activated; duration of 5 to 15 minutes c. Descent phase: Significant increase in contractions; Ferguson reflux activated; average duration varies d. Transitional phase: Woman "laboring down"; fetal station 0; duration of 15 minutes

C The descent phase begins with a significant increase in contractions; the Ferguson reflex is activated, and the duration varies, depending on a number of factors. The latent phase is the lull or "laboring down" period at the beginning of the second stage and lasts 10 to 30 minutes on average. The second stage of labor has no active phase. The transition phase is the final phase in the second stage of labor; contractions are strong and painful.

The first 1 to 2 hours after birth is sometimes referred to as what? a. Bonding period b. Third stage of labor c. Fourth stage of labor d. Early postpartum period

C The first 2 hours of the birth are a critical time for the mother and her baby and is often called the fourth stage of labor. Maternal organs undergo their initial readjustment to a nonpregnant state. The third stage of labor lasts from the birth of the baby to the expulsion of the placenta. Bonding will occur over a much longer period, although it may be initiated during the fourth stage of labor.

Which documentation on a woman's chart on postpartum day 14 indicates a normal involution process? a. Moderate bright red lochial flow b. Breasts firm and tender c. Fundus below the symphysis and nonpalpable d. Episiotomy slightly red and puffy

C The fundus descends 1 cm per day; consequently, it is no longer palpable by postpartum day 14. The lochia should be changed by this day to serosa. Breasts are not part of the involution process. The episiotomy should not be red or puffy at this stage.

A woman in labor has just received an epidural block. What is the most important nursing intervention at this time? a. Limit parenteral fluids. b. Monitor the fetus for possible tachycardia. c. Monitor the maternal blood pressure for possible hypotension. d. Monitor the maternal pulse for possible bradycardia.

C The most important nursing intervention for a woman who has received an epidural block is for the nurse to monitor the maternal blood pressure frequently for signs of hypotension. IV fluids are increased for a woman receiving an epidural to prevent hypotension. The nurse also observes for signs of fetal bradycardia and monitors for signs of maternal tachycardia, secondary to hypotension.

A nulliparous woman has just begun the latent phase of the second stage of her labor. The nurse should anticipate which behavior? a. A nulliparous woman will experience a strong urge to bear down. b. Perineal bulging will show. c. A nulliparous woman will remain quiet with her eyes closed between contractions. d. The amount of bright red bloody show will increase.

C The woman is able to relax and close her eyes between contractions as the fetus passively descends. The woman may be very quiet during this phase. During the latent phase of the second stage of labor, the urge to bear down is often absent or only slight during the acme of the contractions. Perineal bulging occurs during the transition phase of the second stage of labor, not at the beginning of the second stage. An increase in bright red bloody show occurs during the descent phase of the second stage of labor.

The uterine contractions of a woman early in the active phase of labor are assessed by an internal uterine pressure catheter (IUPC). The uterine contractions occur every 3 to 4 minutes and last an average of 55 to 60 seconds. They are becoming more regular and are moderate to strong. Based on this information, what would a prudent nurse do next? a. Immediately notify the woman's primary health care provider. b. Prepare to administer an oxytocic to stimulate uterine activity. c. Document the findings because they reflect the expected contraction pattern for the active phase of labor. d. Prepare the woman for the onset of the second stage of labor.

C The nurse is responsible for monitoring the uterine contractions to ascertain whether they are powerful and frequent enough to accomplish the work of expelling the fetus and the placenta. In addition, the nurse documents these findings in the client's medical record. This labor pattern indicates that the client is in the active phase of the first stage of labor. Nothing indicates a need to notify the primary health care provider at this time. Oxytocin augmentation is not needed for this labor pattern; this contraction pattern indicates that the woman is in active labor. Her contractions will eventually become stronger, last longer, and come closer together during the transition phase of the first stage of labor. The transition phase precedes the second stage of labor, or delivery of the fetus.

Which action is correct when palpation is used to assess the characteristics and pattern of uterine contractions? a. Placing the hand on the abdomen below the umbilicus and palpating uterine tone with the fingertips b. Determining the frequency by timing from the end of one contraction to the end of the next contraction c. Evaluating the intensity by pressing the fingertips into the uterine fundus d. Assessing uterine contractions every 30 minutes throughout the first stage of labor

C The nurse or primary health care provider may assess uterine activity by palpating the fundal section of the uterus using the fingertips. Many women may experience labor pain in the lower segment of the uterus, which may be unrelated to the firmness of the contraction detectable in the uterine fundus. The frequency of uterine contractions is determined by palpating from the beginning of one contraction to the beginning of the next contraction. Assessment of uterine activity is performed in intervals based on the stage of labor. As labor progresses, this assessment is performed more frequently.

A woman has requested an epidural for her pain. She is 5 cm dilated and 100% effaced. The baby is in a vertex position and is engaged. The nurse increases the woman's IV fluid for a preprocedural bolus. The nurse reviews her laboratory values and notes that the woman's hemoglobin is 12 g/dl, hematocrit is 38%, platelets are 67,000, and white blood cells (WBCs) are 12,000/mm3. Which factor would contraindicate an epidural for this woman? a. She is too far dilated. b. She is anemic. c. She has thrombocytopenia. d. She is septic.

C The platelet count indicates a coagulopathy, specifically, thrombocytopenia (low platelets), which is a contraindication to epidural analgesia and anesthesia. Typically, epidural analgesia and anesthesia are used in the laboring woman when a regular labor pattern has been achieved, as evidenced by progressive cervical change. The laboratory values show that the woman's hemoglobin and hematocrit levels are in the normal range and show a slight increase in the WBC count that is not uncommon in laboring women.

Nurses should be cognizant of what regarding the mechanism of labor? a. Seven critical movements must progress in a more or less orderly sequence. b. Asynclitism is sometimes achieved by means of the Leopold's maneuver. c. Effects of the forces determining descent are modified by the shape of the woman's pelvis and the size of the fetal head. d. At birth, the baby is said to achieve "restitution"; that is, a return to the C-shape of the womb.

C The size of the maternal pelvis and the ability of the fetal head to mold also affect the process. The seven identifiable movements of the mechanism of labor simultaneously occur in combinations, not in precise sequences. Asynclitism is the deflection of the baby's head; the Leopold's maneuver is a means of judging descent by palpating the mother's abdomen. Restitution is the rotation of the baby's head after the infant is born.

Under the Newborns' and Mothers' Health Protection Act, all health plans are required to allow new mothers and newborns to remain in the hospital for a minimum of _____ hours after a normal vaginal birth and for _____ hours after a cesarean birth. What is the correct interpretation of this legislation? a. 24; 72 b. 24; 96 c. 48; 96 d. 48; 120

C The specified stays are 48 hours (2 days) for a vaginal birth and 96 hours (4 days) for a cesarean birth. The attending provider and the mother together can decide on an earlier discharge. A client may be discharged either 24 hours after a vaginal birth or 72 hours after a cesarean birth if she is stable and her provider is in agreement. A client is unlikely to remain in the hospital for 120 hours after a cesarean birth unless complications have developed.

What is a distinct advantage of external EFM? a. The ultrasound transducer can accurately measure short-term variability and beat-to-beat changes in the FHR. b. The tocotransducer can measure and record the frequency, regularity, intensity, and approximate duration of uterine contractions. c. The tocotransducer is especially valuable for measuring uterine activity during the first stage of labor. d. Once correctly applied by the nurse, the transducer need not be repositioned even when the woman changes positions.

C The tocotransducer is valuable for measuring uterine activity during the first stage of labor and is especially true when the membranes are intact. Short-term variability and beat-to-beat changes cannot be measured with this technology. The tocotransducer cannot measure and record the intensity of uterine contractions. The transducer must be repositioned when the woman or the fetus changes position.

A 25-year-old gravida 3, para 2 client gave birth to a 9-pound, 7-ounce boy, 4 hours ago after augmentation of labor with oxytocin (Pitocin). She presses her call light, and asks for her nurse right away, stating "I'm bleeding a lot." What is the most likely cause of postpartum hemorrhaging in this client? a. Retained placental fragments b. Unrepaired vaginal lacerations c. Uterine atony d. Puerperal infection

C This woman gave birth to a macrosomic infant after oxytocin augmentation. Combined with these risk factors, uterine atony is the most likely cause of bleeding 4 hours after delivery. Although retained placental fragments may cause postpartum hemorrhaging, it is typically detected within the first hour after delivery of the placenta and is not the most likely cause of the hemorrhaging in this woman. Although unrepaired vaginal lacerations may also cause bleeding, it typically occurs in the period immediately after birth. Puerperal infection can cause subinvolution and subsequent bleeding that is, however, typically detected 24 hours postpartum.

Which statement related to cephalopelvic disproportion (CPD) is the least accurate? a. CPD can be related to either fetal size or fetal position. b. The fetus cannot be born vaginally. c. CPD can be accurately predicted. d. Causes of CPD may have maternal or fetal origins.

C Unfortunately, accurately predicting CPD is not possible. Although CPD is often related to excessive fetal size (macrosomia), malposition of the fetal presenting part is the problem in many cases, not true CPD. When CPD is present, the fetus cannot fit through the maternal pelvis to be born vaginally. CPD may be related to either fetal origins such as macrosomia or malposition or maternal origins such as a too small or malformed pelvis.

Which adaptation of the maternal-fetal exchange of oxygen occurs in response to uterine contraction? a. The maternal-fetal exchange of oxygen and waste products continues except when placental functions are reduced. b. This maternal-fetal exchange increases as the blood pressure decreases. c. It diminishes as the spiral arteries are compressed. d. This exchange of oxygen and waste products is not significantly affected by contractions.

C Uterine contractions during labor tend to decrease circulation through the spiral electrodes and subsequent perfusion through the intervillous space. The maternal blood supply to the placenta gradually stops with contractions. The exchange of oxygen and waste products decreases. The exchange of oxygen and waste products is affected by contractions.

The nurse providing care for a high-risk laboring woman is alert for late FHR decelerations. Which clinical finding might be the cause for these late decelerations? a. Altered cerebral blood flow b. Umbilical cord compression c. Uteroplacental insufficiency d. Meconium fluid

C Uteroplacental insufficiency results in late FHR decelerations. Altered fetal cerebral blood flow results in early FHR decelerations. Umbilical cord compression results in variable FHR decelerations. Meconium-stained fluid may or may not produce changes in the FHR, depending on the gestational age of the fetus and whether other causative factors associated with fetal distress are present.

A woman is having her first child. She has been in labor for 15 hours. A vaginal examination performed 2 hours earlier revealed the cervix to be dilated to 5 cm and 100% effaced, and the presenting part of the fetus was at station 0; however, another vaginal examination performed 5 minutes ago indicated no changes. What abnormal labor pattern is associated with this description? a. Prolonged latent phase b. Protracted active phase c. Secondary arrest d. Protracted descent

C With a secondary arrest of the active phase, the progress of labor has stopped. This client has not had any anticipated cervical change, indicating an arrest of labor. In the nulliparous woman, a prolonged latent phase typically lasts longer than 20 hours. A protracted active phase, the first or second stage of labor, is prolonged (slow dilation). With a protracted descent, the fetus fails to descend at an anticipated rate during the deceleration phase and second stage of labor.

Two days ago a woman gave birth to a full-term infant. Last night she awakened several times to urinate and noted that her gown and bedding were wet from profuse diaphoresis. Which physiologic alteration is the cause for the diaphoresis and diuresis that this client is experiencing? a. Elevated temperature caused by postpartum infection b. Increased basal metabolic rate after giving birth c. Loss of increased blood volume associated with pregnancy d. Increased venous pressure in the lower extremities

C Within 12 hours of birth, women begin to lose the excess tissue fluid that has accumulated during pregnancy. One mechanism for reducing these retained fluids is the profuse diaphoresis that often occurs, especially at night, for the first 2 or 3 days after childbirth. Postpartal diuresis is another mechanism by which the body rids itself of excess fluid. An elevated temperature causes chills and possibly dehydration, not diaphoresis and diuresis. Diaphoresis and diuresis are sometimes referred to as reversal of the water metabolism of pregnancy, not as the basal metabolic rate. Postpartal diuresis may be caused by the removal of increased venous pressure in the lower extremities.

Changes in blood volume after childbirth depend on several factors such as blood loss during childbirth and the amount of extravascular water (physiologic edema) mobilized and excreted. What amount of blood loss does the postpartum nurse anticipate? (Select all that apply.) a. 100 ml b. 250 ml or less c. 300 to 500 ml d. 500 to 1000 ml e. 1500 ml or greater

C, D The average blood loss for a vaginal birth of a single fetus ranges from 300 to 500 ml (10% of blood volume). The typical blood loss for women who gave birth by cesarean is 500 to 1000 ml (15% to 30% of blood volume). During the first few days after childbirth, the plasma volume further decreases as a result of diuresis. Pregnancy-induced hypervolemia (i.e., an increase in blood volume of at least 35%) allows most women to tolerate considerable blood loss during childbirth.

Which definition of an acceleration in the fetal heart rate (FHR) is accurate? a. FHR accelerations are indications of fetal well-being when they are periodic. b. FHR accelerations are greater and longer in preterm gestations. c. FHR accelerations are usually observed with breech presentations when they are episodic. d. An acceleration in the FHR presents a visually apparent and abrupt peak.

D Acceleration of the FHR is defined as a visually apparent abrupt (only to peak 30 seconds) increase in the FHR above the baseline rate. Periodic accelerations occur with uterine contractions and are usually observed with breech presentations. Episodic accelerations occur during fetal movement and are indications of fetal well-being. Preterm accelerations peak at 10 beats per minute above the baseline and last for at least 10 seconds.

A 25-year-old gravida 1 para 1 who had an emergency cesarean birth 3 days ago is scheduled for discharge. As the nurse prepares her for discharge, she begins to cry. The nurse's next action should be what? a. Assess her for pain. b. Point out how lucky she is to have a healthy baby. c. Explain that she is experiencing postpartum blues. d. Allow her time to express her feelings.

D Although many women experience transient postpartum blues, they need assistance in expressing their feelings. Postpartum blues affects 50% to 80% of new mothers. An assumption that the client is in pain should not be made when, in fact, she may have no pain whatsoever. Making this assumption would be blocking communication and inappropriate in this situation. The client needs the opportunity to express her feelings first; client teaching can occur later.

The nurse is providing instruction to the newly delivered client regarding postbirth uterine and vaginal discharge, called lochia. Which statement is the most appropriate? a. Lochia is similar to a light menstrual period for the first 6 to 12 hours. b. It is usually greater after cesarean births. c. Lochia will usually decrease with ambulation and breastfeeding. d. It should smell like normal menstrual flow unless an infection is present.

D An offensive odor usually indicates an infection. Lochia flow should approximate a heavy menstrual period for the first 2 hours and then steadily decrease. Less lochia is usually seen after cesarean births and usually increases with ambulation and breastfeeding.

The obstetric nurse is preparing the client for an emergency cesarean birth, with no time to administer spinal anesthesia. The nurse is aware of and prepared for the greatest risk of administering general anesthesia to the client. What is this risk? a. Respiratory depression b. Uterine relaxation c. Inadequate muscle relaxation d. Aspiration of stomach contents

D Aspiration of acidic gastric contents with possible airway obstruction is a potentially fatal complication of general anesthesia. Respirations can be altered during general anesthesia, and the anesthesiologist will take precautions to maintain proper oxygenation. Uterine relaxation can occur with some anesthesia but can be monitored and prevented. Inadequate muscle relaxation can be improved with medication.

When a nulliparous woman telephones the hospital to report that she is in labor, what guidance should the nurse provide or information should the nurse obtain? a. Tell the woman to stay home until her membranes rupture. b. Emphasize that food and fluid intake should stop. c. Arrange for the woman to come to the hospital for labor evaluation. d. Ask the woman to describe why she believes she is in labor.

D Assessment begins at the first contact with the woman, whether by telephone or in person. By asking the woman to describe her signs and symptoms, the nurse can begin her assessment and gather data. The initial nursing activity should be to gather data about the woman's status. The amniotic membranes may or may not spontaneously rupture during labor. The client may be instructed to stay home until the uterine contractions become strong and regular. Before instructing the woman to come to the hospital, the nurse should initiate her assessment during the telephone interview. After this assessment has been made, the nurse may want to discuss the appropriate oral intake for early labor, such as light foods or clear liquids, depending on the preference of the client or her primary health care provider.

A client is concerned that her breasts are engorged and uncomfortable. What is the nurse's explanation for this physiologic change? a. Overproduction of colostrum b. Accumulation of milk in the lactiferous ducts and glands c. Hyperplasia of mammary tissue d. Congestion of veins and lymphatic vessels

D Breast engorgement is caused by the temporary congestion of veins and lymphatic vessels. An overproduction of colostrum, an accumulation of milk in the lactiferous ducts and glands, and hyperplasia of mammary tissue do not cause breast engorgement.

Parents who have not already done so need to make time for newborn follow-up of the discharge. According to the American Academy of Pediatrics (AAP), when should a breastfeeding infant first need to be seen for a follow-up examination? a. 2 weeks of age b. 7 to 10 days after childbirth c. 4 to 5 days after hospital discharge d. 48 to 72 hours after hospital discharge

D Breastfeeding infants are routinely seen by the pediatric health care provider clinic within 3 to 5 days after birth or 48 to 72 hours after hospital discharge and again at 2 weeks of age. Formula-feeding infants may be seen for the first time at 2 weeks of age.

A woman who is pregnant for the first time is dilated 3 cm and having contractions every 5 minutes. She is groaning and perspiring excessively; she states that she did not attend childbirth classes. What is the optimal intervention for the nurse to provide at this time? a. Notify the woman's health care provider. b. Administer the prescribed narcotic analgesic. c. Assure her that her labor will be over soon. d. Assist her with simple breathing and relaxation instructions.

D By reducing tension and stress, both focusing and relaxation techniques will allow the woman in labor to rest and conserve energy for the task of giving birth. For those who have had no preparation, instruction in simple breathing and relaxation can be given in early labor and is often successful. The nurse can independently perform many functions in labor and birth, such as teaching and support. Pain medication may be an option for this client. However, the initial response of the nurse should include teaching the client about her options. The length of labor varies among individuals, but the first stage of labor is the longest. At 3 cm of dilation with contractions every 5 minutes, this woman has a significant amount of labor yet to experience.

While providing routine mother-baby care, which activities should the nurse encourage to facilitate the parent-infant attachment? a. The baby is able to return to the nursery at night so that the new mother can sleep. b. Routine times for care are established to reassure the parents. c. The father should be encouraged to go home at night to prepare for discharge of the mother and baby. d. An environment that fosters as much privacy as possible should be created.

D Care providers need to knock before gaining entry. Nursing care activities should be grouped. Once the baby has demonstrated an adjustment to extrauterine life (either in the mother's room or the transitional nursery), all care should be provided in one location. This important principle of family-centered maternity care fosters attachment by offering parents the opportunity to learn about their infant 24 hours a day. One nurse should provide care to both mother and baby in this couplet care or rooming-in model. It is not necessary for the baby to return to the nursery at night. In fact, the mother will sleep better with the infant close by. Care should be individualized to meet the parents' needs, not the routines of the staff. Teaching goals should be developed in collaboration with the parents. The father or significant other should be permitted to sleep in the room with the mother. The maternity unit should develop policies that allow for the presence of significant others as much as the new mother desires.

A woman at 26 weeks of gestation is being assessed to determine whether she is experiencing preterm labor. Which finding indicates that preterm labor is occurring? a. Estriol is not found in maternal saliva. b. Irregular, mild uterine contractions are occurring every 12 to 15 minutes. c. Fetal fibronectin is present in vaginal secretions. d. The cervix is effacing and dilated to 2 cm.

D Cervical changes such as shortened endocervical length, effacement, and dilation are predictors of imminent preterm labor. Changes in the cervix accompanied by regular contractions indicate labor at any gestation. Estriol is a form of estrogen produced by the fetus that is present in plasma at 9 weeks of gestation. Levels of salivary estriol have been shown to increase before preterm birth. Irregular, mild contractions that do not cause cervical change are not considered a threat. The presence of fetal fibronectin in vaginal secretions between 24 and 36 weeks of gestation could predict preterm labor, but it has only a 20% to 40% positive predictive value. Of more importance are other physiologic clues of preterm labor such as cervical changes.

An 18-year-old pregnant woman, gravida 1, para 0, is admitted to the labor and birth unit with moderate contractions every 5 minutes that last 40 seconds. The client states, "My contractions are so strong, I don't know what to do." Before making a plan of care, what should the nurse's first action be? a. Assess for fetal well-being. b. Encourage the woman to lie on her side. c. Disturb the woman as little as possible. d. Recognize that pain is personalized for each individual

D Each woman's pain during childbirth is unique and is influenced by a variety of physiologic, psychosocial, and environmental factors. A critical issue for the nurse is how support can make a difference in the pain of the woman during labor and birth. This scenario includes no information that would indicate fetal distress or a logical reason to be overly concerned about the well-being of the fetus. The left lateral position is used to alleviate fetal distress, not maternal stress. The nurse has an obligation to provide physical, emotional, and psychosocial care and support to the laboring woman. This client clearly needs support.

What physiologic change occurs as the result of increasing the infusion rate of nonadditive IV fluids? a. Maintaining normal maternal temperature b. Preventing normal maternal hypoglycemia c. Increasing the oxygen-carrying capacity of the maternal blood d. Expanding maternal blood volume

D Filling the mother's vascular system increases the amount of blood available to perfuse the placenta and may correct hypotension. Increasing fluid volume may alter the maternal temperature only if she is dehydrated. Most IV fluids for laboring women are isotonic and do not provide extra glucose. Oxygen-carrying capacity is increased by adding more red blood cells.

The nurse is performing an assessment on a client who thinks she may be experiencing preterm labor. Which information is the most important for the nurse to understand and share with the client? a. Because all women must be considered at risk for preterm labor and prediction is so variable, teaching pregnant women the symptoms of preterm labor probably causes more harm through false alarms. b. Braxton Hicks contractions often signal the onset of preterm labor. c. Because preterm labor is likely to be the start of an extended labor, a woman with symptoms can wait several hours before contacting the primary caregiver. d. Diagnosis of preterm labor is based on gestational age, uterine activity, and progressive cervical change.

D Gestational age of 20 to 37 weeks, uterine contractions, and a cervix that is 80% effaced or dilated 2 cm indicates preterm labor. It is essential that nurses teach women how to detect the early symptoms of preterm labor. Braxton Hicks contractions resemble preterm labor contractions, but they are not true labor. Waiting too long to see a health care provider could result in essential medications failing to be administered. Preterm labor is not necessarily long-term labor.

After giving birth to a healthy infant boy, a primiparous client, 16 years of age, is admitted to the postpartum unit. An appropriate nursing diagnosis for her at this time is "Deficient knowledge of infant care." What should the nurse be certain to include in the plan of care as he or she prepares the client for discharge? a. Teach the client how to feed and bathe her infant. b. Give the client written information on bathing her infant. c. Advise the client that all mothers instinctively know how to care for their infants. d. Provide time for the client to bathe her infant after she views a demonstration of infant bathing.

D Having the mother demonstrate infant care is a valuable method of assessing the client's understanding of her newly acquired knowledge, especially in this age group, because she may inadvertently neglect her child. Although verbalizing how to care for the infant is a form of client education or providing written information might be useful, neither is the most developmentally appropriate teaching method for a teenage mother. Advising the young woman that all mothers instinctively know how to care for their infants is inappropriate; it is belittling and false.

Which condition, not uncommon in pregnancy, is likely to require careful medical assessment during the puerperium? a. Varicosities of the legs b. Carpal tunnel syndrome c. Periodic numbness and tingling of the fingers d. Headaches

D Headaches in the postpartum period can have a number of causes, some of which deserve medical attention. Total or nearly total regression of varicosities is expected after childbirth. Carpal tunnel syndrome is relieved in childbirth when the compression on the median nerve is lessened. Periodic numbness of the fingers usually disappears after childbirth unless carrying the baby aggravates the condition.

Which statement correctly describes the effects of various pain factors? a. Higher prostaglandin levels arising from dysmenorrhea can blunt the pain of childbirth. b. Upright positions in labor increase the pain factor because they cause greater fatigue. c. Women who move around trying different positions experience more pain. d. Levels of pain-mitigating beta-endorphins are higher during a spontaneous, natural childbirth.

D Higher endorphin levels help women tolerate pain and reduce anxiety and irritability. Higher prostaglandin levels correspond to more severe labor pains. Upright positions in labor usually result in improved comfort and less pain. Moving freely to find more comfortable positions is important for reducing pain and muscle tension.

Which statement related to the induction of labor is most accurate? a. Can be achieved by external and internal version techniques b. Is also known as a trial of labor (TOL) c. Is almost always performed for medical reasons d. Is rated for viability by a Bishop score

D Induction of labor is likely to be more successful with a Bishop score of 9 or higher for first-time mothers or 5 or higher for veterans. Version is the turning of the fetus to a better position by a physician for an easier or safer birth. A TOL is the observance of a woman and her fetus for several hours of active labor to assess the safety of vaginal birth. Two thirds of cases of induced labor are elective and not done for medical reasons.

Because a full bladder prevents the uterus from contracting normally, nurses intervene to help the woman spontaneously empty her bladder as soon as possible. If all else fails, what tactic might the nurse use? a. Pouring water from a squeeze bottle over the woman's perineum b. Placing oil of peppermint in a bedpan under the woman c. Asking the physician to prescribe analgesic agents d. Inserting a sterile catheter

D Invasive procedures are usually the last to be tried, especially with so many other simple and easy methods available (e.g., water, peppermint vapors, pain pills). Pouring water over the perineum may stimulate voiding. It is easy, noninvasive, and should be tried first. The oil of peppermint releases vapors that may relax the necessary muscles. It, too, is easy, noninvasive, and should be tried early on. If the woman is anticipating pain from voiding, then pain medications may be helpful. Other nonmedical means should be tried first, but medications still come before the insertion of a catheter.

Which FHR finding is the most concerning to the nurse who is providing care to a laboring client? a. Accelerations with fetal movement b. Early decelerations c. Average FHR of 126 beats per minute d. Late decelerations

D Late decelerations are caused by uteroplacental insufficiency and are associated with fetal hypoxemia. Late FHR decelerations are considered ominous if they are persistent and left uncorrected. Accelerations with fetal movement are an indication of fetal well-being. Early decelerations in the FHR are associated with head compression as the fetus descends into the maternal pelvic outlet; they are not generally a concern during normal labor. An FHR finding of 126 beats per minute is normal and not a concern.

The nurse is using intermittent auscultation (IA) to locate the fetal heartbeat. Which statement regarding this method of surveillance is accurate? a. The nurse can be expected to cover only two or three clients when IA is the primary method of fetal assessment. b. The best course is to use the descriptive terms associated with EFM when documenting results. c. If the heartbeat cannot be immediately found, then a shift must be made to EFM. d. Ultrasound can be used to find the FHR and to reassure the mother if the initial difficulty is a factor.

D Locating fetal heartbeats often takes time. Mothers can be verbally reassured and reassured by viewing the ultrasound pictures if that device is used to help locate the heartbeat. When used as the primary method of fetal assessment, IA requires a nurse-to-client ratio of one to one. Documentation should use only terms that can be numerically defined; the usual visual descriptions of EFM are inappropriate.

A woman gave birth to a healthy infant boy 5 days ago. What type of lochia does the nurse expect to find when evaluating this client? a. Lochia rubra b. Lochia sangra c. Lochia alba d. Lochia serosa

D Lochia serosa, which consists of blood, serum, leukocytes, and tissue debris, generally occurs around day 3 or 4 after childbirth. Lochia rubra consists of blood and decidual and trophoblastic debris. The flow generally lasts 3 to 4 days and pales, becoming pink or brown. Lochia sangra is not a real term. Lochia alba occurs in most women after day 10 and can continue up to 6 weeks after childbirth.

The perinatal nurse realizes that an FHR that is tachycardic, bradycardic, has late decelerations, or loss of variability is nonreassuring and is associated with which condition? a. Hypotension b. Cord compression c. Maternal drug use d. Hypoxemia

D Nonreassuring FHR patterns are associated with fetal hypoxemia. Fetal bradycardia may be associated with maternal hypotension. Variable FHR decelerations are associated with cord compression. Maternal drug use is associated with fetal tachycardia.

The trend in the United States is for women to remain hospitalized no longer than 1 or 2 days after giving birth. Which scenario is not a contributor to this model of care? a. Wellness orientation model of care rather than a sick-care model b. Desire to reduce health care costs c. Consumer demand for fewer medical interventions and more family-focused experiences d. Less need for nursing time as a result of more medical and technologic advances and devices available at home that can provide information

D Nursing time and care are in demand as much as ever; the nurse simply has to do things more quickly. A wellness orientation model of care seems to focus on getting clients out the door sooner. In most cases, less hospitalization results in lower costs. People believe that the family gives more nurturing care than the institution.

A 25-year-old multiparous woman gave birth to an infant boy 1 day ago. Today her husband brings a large container of brown seaweed soup to the hospital. When the nurse enters the room, the husband asks for help with warming the soup so that his wife can eat it. What is the nurse's most appropriate response? a. "Didn't you like your lunch?" b. "Does your physician know that you are planning to eat that?" c. "What is that anyway?" d. "I'll warm the soup in the microwave for you."

D Offering to warm the food shows cultural sensitivity to the dietary preferences of the woman and is the most appropriate response. Cultural dietary preferences must be respected. Women may request that family members bring favorite or culturally appropriate foods to the hospital. Asking the woman to identify her food does not show cultural sensitivity.

In follow-up appointments or visits with parents and their new baby, it is useful if the nurse can identify infant behaviors that can either facilitate or inhibit attachment. What is an inhibiting behavior? a. The infant cries only when hungry or wet. b. The infant's activity is somewhat predictable. c. The infant clings to the parents. d. The infant seeks attention from any adult in the room.

D Parents want to be the focus of the infant's existence, just as the infant is the focus of their existence. Facilitating and inhibiting behaviors build or discourage bonding (attitudes); they do not reflect any value judgments on what might be healthy or unhealthy. The infant who shows no preference for his or her parents over other adults is exhibiting an inhibiting behavior. An infant who cries only when hungry or wet is exhibiting a facilitating behavior. An infant who has a predictable attention span is exhibiting a facilitating behavior. The infant who clings to his or her parents, enjoys being cuddled and held, and is easily consoled is displaying facilitating behaviors.

Which assessment is least likely to be associated with a breech presentation? a. Meconium-stained amniotic fluid b. Fetal heart tones heard at or above the maternal umbilicus c. Preterm labor and birth d. Postterm gestation

D Postterm gestation is not likely to occur with a breech presentation. The presence of meconium in a breech presentation may be a result of pressure on the fetal wall as it traverses the birth canal. Fetal heart tones heard at the level of the umbilical level of the mother are a typical finding in a breech presentation because the fetal back would be located in the upper abdominal area. Breech presentations often occur in preterm births.

Which collection of risk factors will most likely result in damaging lacerations, including episiotomies? a. Dark-skinned woman who has had more than one pregnancy, who is going through prolonged second-stage labor, and who is attended by a midwife b. Reddish-haired mother of two who is going through a breech birth c. Dark-skinned first-time mother who is going through a long labor d. First-time mother with reddish hair whose rapid labor was overseen by an obstetrician

D Reddish-haired women have tissue that is less distensible than darker-skinned women and therefore may have less efficient healing. First-time mothers are also at greater risk, especially with breech births, long second-stage labors, or rapid labors during which the time for the perineum to stretch is insufficient. The rate of episiotomies is higher when obstetricians rather than midwives attend the births. The woman in the first scenario (a) is at low risk for either damaging lacerations or an episiotomy. She is multiparous, has dark skin, and is being attended by a midwife, who is less likely to perform an episiotomy. Reddish-haired women have tissue that is less distensible than that of darker-skinned women. Consequently, the client in the second scenario (b) is at increased risk for lacerations; however, she has had two previous deliveries, which result in a lower likelihood of an episiotomy. The fact that the woman in the third scenario (c) is experiencing a prolonged labor might increase her risk for lacerations. Fortunately, she is dark skinned, which indicates that her tissue is more distensible than that of fair-skinned women and therefore less susceptible to injury.

The obstetric provider has informed the nurse that she will be performing an amniotomy on the client to induce labor. What is the nurse's highest priority intervention after the amniotomy is performed? a. Applying clean linens under the woman b. Taking the client's vital signs c. Performing a vaginal examination d. Assessing the fetal heart rate (FHR)

D The FHR is assessed before and immediately after the amniotomy to detect any changes that might indicate cord compression or prolapse. Providing comfort measures, such as clean linens, for the client is important but not the priority immediately after an amniotomy. The woman's temperature should be checked every 2 hours after the rupture of membranes but not the priority immediately after an amniotomy. The woman would have had a vaginal examination during the procedure. Unless cord prolapse is suspected, another vaginal examination is not warranted. Additionally, FHR assessment provides clinical cues to a prolapsed cord.

Which statement regarding the care of a client in labor is correct and important to the nurse as he or she formulates the plan of care? a. The woman's blood pressure will increase during contractions and fall back to prelabor normal levels between contractions. b. The use of the Valsalva maneuver is encouraged during the second stage of labor to relieve fetal hypoxia. c. Having the woman point her toes will reduce leg cramps. d. Endogenous endorphins released during labor will raise the woman's pain threshold and produce sedation.

D The endogenous endorphins released during labor will raise the woman's pain threshold and produce sedation. In addition, physiologic anesthesia of the perineal tissues, caused by the pressure of the presenting part, decreases the mother's perception of pain. Blood pressure levels increase during contractions but remain somewhat elevated between them. The use of the Valsalva maneuver is discouraged during the second stage labor because of a number of unhealthy outcomes, including fetal hypoxia. Pointing the toes can cause leg cramps, as can the process of labor itself.

A client asks the nurse when her ovaries will begin working again. Which explanation by the nurse is most accurate? a. Almost 75% of women who do not breastfeed resume menstruating within 1 month after birth. b. Ovulation occurs slightly earlier for breastfeeding women. c. Because of menstruation and ovulation schedules, contraception considerations can be postponed until after the puerperium. d. The first menstrual flow after childbirth usually is heavier than normal.

D The first flow is heavier, but within three or four cycles, the flow is back to normal. Ovulation can occur within the first month, but for 70% of nonlactating women, it returns in approximately 3 months. Women who are breastfeeding take longer to resume ovulation. Because many women ovulate before their first postpartum menstrual period, contraceptive options need to be discussed early in the puerperium.

A new mother asks the nurse when the "soft spot" on her son's head will go away. What is the nurse's best response, based upon her understanding of when the anterior frontal closes? a. 2 months b. 8 months c. 12 months d. 18 months

D The larger of the two fontanels, the anterior fontanel, closes by 18 months after birth. The posterior fontanel closes at 6 to 8 weeks. The remaining three options are too early for the anterior fontanel to close.

After birth, a crying infant may be soothed by being held in a position in which the newborn can hear the mother's heartbeat. This phenomenon is known as what? a. Entrainment b. Reciprocity c. Synchrony d. Biorhythmicity

D The newborn is in rhythm with the mother. The infant develops a personal biorhythm with the parents' help over time. Entrainment is the movement of a newborn in time to the structure of adult speech. Reciprocity is body movement or behavior that gives cues to the person's desires. These take several weeks to develop with a new baby. Synchrony is the fit between the infant's behavioral cues and the parent's responses.

A new father states, "I know nothing about babies"; however, he seems to be interested in learning. How would the nurse best respond to this father? a. Continue to observe his interaction with the newborn. b. Tell him when he does something wrong. c. Show no concern; he will learn on his own. d. Include him in teaching sessions.

D The nurse must be sensitive to the father's needs and include him whenever possible. As fathers take on their new role, the nurse should praise every attempt, even if his early care is awkward. Although noting the bonding process of the mother and the father is important, it does not satisfy the expressed needs of the father. The new father should be encouraged to care for his baby by pointing out the things that he does right. Criticizing him will discourage him.

The nurse performs a vaginal examination to assess a client's labor progress. Which action should the nurse take next? a. Perform an examination at least once every hour during the active phase of labor. b. Perform the examination with the woman in the supine position. c. Wear two clean gloves for each examination. d. Discuss the findings with the woman and her partner.

D The nurse should discuss the findings of the vaginal examination with the woman and her partner, as well as report the findings to the primary care provider. A vaginal examination should be performed only when indicated by the status of the woman and her fetus. The woman should be positioned so as to avoid supine hypotension. The examiner should wear a sterile glove while performing a vaginal examination for a laboring woman.

On examining a woman who gave birth 5 hours ago, the nurse finds that the woman has completely saturated a perineal pad within 15 minutes. What is the nurse's highest priority at this time? a. Beginning an intravenous (IV) infusion of Ringer's lactate solution b. Assessing the woman's vital signs c. Calling the woman's primary health care provider d. Massaging the woman's fundus

D The nurse should first assess the uterus for atony by massaging the woman's fundus. Uterine tone must be established to prevent excessive blood loss. The nurse may begin an IV infusion to restore circulatory volume, but this would not be the first action. Blood pressure is not a reliable indicator of impending shock from impending hemorrhage; assessing vital signs should not be the nurse's first action. The physician would be notified after the nurse completes the assessment of the woman.

A 30-year-old multiparous woman has a boy who is years old and has recently delivered an infant girl. She tells the nurse, "I don't know how I'll ever manage both children when I get home." Which suggestion would assist this new mother in alleviating sibling rivalry? a. Tell the older child that he is a big boy now and should love his new sister. b. Let the older child stay with his grandparents for the first 6 weeks to allow him to adjust to the newborn. c. Ask friends and relatives not to bring gifts to the older sibling because you do not want to spoil him. d. Realize that the regression in habits and behaviors in the older child is a typical reaction and that he needs extra love and attention at this time.

D The older child may regress in habits or behaviors (e.g., toileting, sleep habits) as a method of seeking attention. Parents need to distribute their attention in an equitable manner. Telling the older child that he should love his new sister is a negative approach to facilitating sibling acceptance of the new infant. Reactions of siblings may result from temporary separation from the mother. Removing the older child from the home when the new infant arrives may enhance negative behaviors from the older child caused by a separation from the mother. Providing small gifts from the infant to the older child is a strategy for facilitating sibling acceptance of the new infant.

Which term best describes the interval between the birth of the newborn and the return of the reproductive organs to their normal nonpregnant state? a. Involutionary period because of what happens to the uterus b. Lochia period because of the nature of the vaginal discharge c. Mini-tri period because it lasts only 3 to 6 weeks d. Puerperium, or fourth trimester of pregnancy

D The puerperium, also called the fourth trimester or the postpartum period of pregnancy, is the final period of pregnancy and lasts approximately 3 to 6 weeks. Involution marks the end of the puerperium. Lochia refers to the various vaginal discharges during the puerperium.

In which situation would the nurse be called on to stimulate the fetal scalp? a. As part of fetal scalp blood sampling b. In response to tocolysis c. In preparation for fetal oxygen saturation monitoring d. To elicit an acceleration in the FHR

D The scalp can be stimulated using digital pressure during a vaginal examination. Fetal scalp blood sampling involves swabbing the scalp with disinfectant before a sample is collected. The nurse stimulates the fetal scalp to elicit an acceleration of the FHR. Tocolysis is relaxation of the uterus. Fetal oxygen saturation monitoring involves the insertion of a sensor.

In evaluating the effectiveness of magnesium sulfate for the treatment of preterm labor, which finding alerts the nurse to possible side effects? a. Urine output of 160 ml in 4 hours b. DTRs 2+ and no clonus c. Respiratory rate (RR) of 16 breaths per minute d. Serum magnesium level of 10 mg/dl

D The therapeutic range for magnesium sulfate management is 4 to 7.5 mg/dl. A serum magnesium level of 10 mg/dl could lead to signs and symptoms of magnesium toxicity, including oliguria and respiratory distress. Urine output of 160 ml in 4 hours, DTRs of 2+, and a RR of 16 breaths per minute are all normal findings.

A multiparous woman has been in labor for 8 hours. Her membranes have just ruptured. What is the nurse's highest priority in this situation? a. Prepare the woman for imminent birth. b. Notify the woman's primary health care provider. c. Document the characteristics of the fluid. d. Assess the fetal heart rate (FHR) and pattern.

D The umbilical cord may prolapse when the membranes rupture. The FHR and pattern should be closely monitored for several minutes immediately after the ROM to ascertain fetal well-being, and the findings should be documented. The ROM may increase the intensity and frequency of the uterine contractions, but it does not indicate that birth is imminent. The nurse may notify the primary health care provider after ROM occurs and the fetal well-being and response to ROM have been assessed. The nurse's priority is to assess fetal well-being. The nurse should document the characteristics of the amniotic fluid, but the initial response is to assess fetal well-being and the response to ROM.

Pelvic floor exercises, also known as Kegel exercises, will help to strengthen the perineal muscles and encourage healing after childbirth. The nurse requests the client to repeat back instructions for this exercise. Which response by the client indicates successful learning? a. "I contract my thighs, buttocks, and abdomen." b. "I perform 10 of these exercises every day." c. "I stand while practicing this new exercise routine." d. "I pretend that I am trying to stop the flow of urine in midstream."

D The woman can pretend that she is attempting to stop the passing of gas or the flow of urine midstream, which will replicate the sensation of the muscles drawing upward and inward. Each contraction should be as intense as possible without contracting the abdomen, buttocks, or thighs. Guidelines suggest that these exercises should be performed 24 to 100 times per day. Positive results are shown with a minimum of 24 to 45 repetitions per day. The best position to learn Kegel exercises is to lie supine with the knees bent. A secondary position is on the hands and knees.

A woman gave birth vaginally to a 9-pound, 12-ounce girl yesterday. Her primary health care provider has written orders for perineal ice packs, use of a sitz bath three times daily, and a stool softener. Which information regarding the client's condition is most closely correlated with these orders? a. Woman is a gravida 2, para 2. b. Woman had a vacuum-assisted birth. c. Woman received epidural anesthesia. d. Woman has an episiotomy.

D These orders are typical interventions for a woman who has had an episiotomy, lacerations, and hemorrhoids. A multiparous classification is not an indication for these orders. A vacuum-assisted birth may be used in conjunction with an episiotomy, which would indicate these interventions. The use of an epidural anesthesia has no correlation with these orders.

A primigravida at 39 weeks of gestation is observed for 2 hours in the intrapartum unit. The FHR has been normal. Contractions are 5 to 9 minutes apart, 20 to 30 seconds in duration, and of mild intensity. Cervical dilation is 1 to 2 cm and uneffaced (unchanged from admission). Membranes are intact. What disposition would the nurse anticipate? a. Admitted and prepared for a cesarean birth b. Admitted for extended observation c. Discharged home with a sedative d. Discharged home to await the onset of true labor

D This situation describes a woman with normal assessments who is probably in false labor and will likely not deliver rapidly once true labor begins. No further assessments or observations are indicated; therefore, the client will be discharged along with instructions to return when contractions increase in intensity and frequency. Neither a cesarean birth nor a sedative is required at this time.

A woman in the active phase of the first stage of labor is using a shallow pattern of breathing, which is approximately twice the normal adult breathing rate. She starts to complain about feeling lightheaded and dizzy and states that her fingers are tingling. Which intervention should the nurse immediately initiate? a. Contact the woman's physician. b. Tell the woman to slow her pace of her breathing. c. Administer oxygen via a mask or nasal cannula. d. Help her breathe into a paper bag.

D This woman is experiencing the side effects of hyperventilation, which include the symptoms of lightheadedness, dizziness, tingling of the fingers, or circumoral numbness. Having the woman breathe into a paper bag held tightly around her mouth and nose may eliminate respiratory alkalosis and enable her to rebreathe carbon dioxide and replace the bicarbonate ion.

A client is in early labor, and her nurse is discussing the pain relief options she is considering. The client states that she wants an epidural "no matter what!" What is the nurse's best response? a. "I'll make sure you get your epidural." b. "You may only have an epidural if your physician allows it." c. "You may only have an epidural if you are going to deliver vaginally." d. "The type of analgesia or anesthesia used is determined, in part, by the stage of your labor and the method of birth."

D To avoid suppressing the progress of labor, pharmacologic measures for pain relief are generally not implemented until labor has advanced to the active phase of the first stage and the cervix is dilated approximately 4 to 5 cm. A plan of care is developed for each woman that addresses her particular clinical and nursing problems. The nurse collaborates with the primary health care provider and the laboring woman in selecting features of care relevant to the woman and her family. The decision whether to use an epidural to relieve labor pain is multifactorial. The nurse should not make a blanket statement guaranteeing the client one pharmacologic option over another until a complete history and physical examination has been obtained. A physician's order is required for pharmacologic options for pain management. However, expressing this requirement is not the nurse's best response. An epidural is an effective pharmacologic pain management option for many laboring women. It can also be used for anesthesia control if the woman undergoes an operative delivery.

The nurse is evaluating the EFM tracing of the client who is in active labor. Suddenly, the FHR drops from its baseline of 125 down to 80 beats per minute. The mother is repositioned, and the nurse provides oxygen, increased IV fluids, and performs a vaginal examination. The cervix has not changed. Five minutes have passed, and the FHR remains in the 80s. What additional nursing measures should the nurse take next? a. Call for help. b. Insert a Foley catheter. c. Start administering Pitocin. d. Immediately notify the care provider.

D To relieve an FHR deceleration, the nurse can reposition the mother, increase IV fluids, and provide oxygen. If oxytocin is infusing, then it should be discontinued. If the FHR does not resolve, then the primary care provider should be immediately notified. Inserting a Foley catheter is an inappropriate nursing action. If the FHR were to continue in a nonreassuring pattern, then a cesarean section could be warranted, which would require a Foley catheter. However, the physician must make that determination. The administration of Pitocin may place additional stress on the fetus.

Maternity nurses often have to answer questions about the many, sometimes unusual, ways people have tried to make the birthing experience more comfortable. Which information regarding nonpharmacologic pain relief is accurate? a. Music supplied by the support person has to be discouraged because it could disturb others or upset the hospital routine. b. Women in labor can benefit from sitting in a bathtub, but they must limit immersion to no longer than 15 minutes at a time. c. Effleurage is permissible, but counterpressure is almost always counterproductive. d. Electrodes attached to either side of the spine to provide high-intensity electrical impulses facilitate the release of endorphins.

D Transcutaneous electrical nerve stimulation (TENS) may help and is most useful for lower back pain that occurs during the first stage of labor. Music may be very helpful for reducing tension and certainly can be accommodated by the hospital. Women can stay in a bath as long as they want, although repeated baths with breaks might be more effective than one long bath. Counterpressure can help the woman cope with lower back pain.

Which nursing action is most appropriate to correct a boggy uterus that is displaced above and to the right of the umbilicus? a. Notify the physician of an impending hemorrhage. b. Assess the blood pressure and pulse. c. Evaluate the lochia. d. Assist the client in emptying her bladder.

D Urinary retention may cause overdistention of the urinary bladder, which lifts and displaces the uterus. Nursing actions need to be implemented before notifying the physician. Evaluating blood pressure, pulse, and lochia is important if the bleeding continues; however, the focus at this point is to assist the client in emptying her bladder.

Which characteristic of a uterine contraction is not routinely documented? a. Frequency: how often contractions occur b. Intensity: strength of the contraction at its peak c. Resting tone: tension in the uterine muscle d. Appearance: shape and height

D Uterine contractions are described in terms of frequency, intensity, duration, and resting tone. Appearance is not routinely charted.

A pregnant woman is at 38 weeks of gestation. She wants to know whether there are any signs that "labor is getting close to starting." Which finding is an indication that labor may begin soon? a. Weight gain of 1.5 to 2 kg (3 to 4 lb) b. Increase in fundal height c. Urinary retention d. Surge of energy

D Women speak of having a burst of energy before labor. The woman may lose 0.5 to 1.5 kg, as a result of water loss caused by electrolyte shifts that, in turn, are caused by changes in the estrogen and progesterone levels. When the fetus descends into the true pelvis (called lightening), the fundal height may decrease. Urinary frequency may return before labor.


Related study sets

Chapter 36: Emergency Preparedness and Protective Practices

View Set

Module 06 Securing Cloud Resources

View Set

OB Ch 27 Care of the Mother and Newborn

View Set

Chapter 19 Analysis and Monitoring of Gas Exchange

View Set

Module 8 - Energy Balance, Weight Management, & Eating Disorders

View Set